Vous êtes sur la page 1sur 28

Classification of property Movable Lopez v.

Orosa FACTS: -Petitioner Lopez was engaged in doing business under the trade name Lopez-Castelo Sawmill. Orosa, a resident of the same province as Lopez, invited the latter to make an investment in the theatre business. Lopez declined to invest but agreed to supply the lumber necessary for the construction of the proposed theatre. They had an oral agreement that Orosa would be personally liable for any account that the said construction might incur and that payment would be on demand and not cash on delivery basis. Lopez delivered the which was used for construction amounting to P62,255.85. He was paid only P20,848.50, leaving a balance of P41,771.35. The land on which the building was erected previously owned by Orosa, was later on acquired by the corporation. . As Lopez was pressing Orosa for payment, the latter and president of the corporation promised to obtain a bank loan by mortgaging the properties of the Plaza Theatre., out of which the unpaid balance would be satisfied. But unknown to Lopez, the corporation already obtained a loan with Luzon Surety Company as surety, and the corporation in turn executed a mortgage on the land and building in favor of the said company as counter-security. Due to the persistent demands of Lopez, Orosa executed a deed of assignment over his shares of stock in the corporation. As it remained unsettled, Lopez filed a case against Orosa and Plaza theatre praying that they be sentenced to pay him jointly and severally of the unpaid balance; and in case defendants fail to pay, the land and building owned by the corporation be sold in public auction with the proceeds be applied to the balance; or the shares of stock be sold in public auction. The lower court held that defendants were jointly liable for the unpaid balance and Lopez thus acquired the material mans lien over the construction. The lien was merely confined to the building and did not extend to the on which the construction was made. Lopez tried to secure a modification of the decision, but was denied. ISSUES: Whether the material mans lien for the value of the materials used in the construction of the building attaches to said structure alone and doesnt extend to the land on which the building is adhered to. Whether the lower court and CA erred in not providing that the material mans liens is superior to the mortgage executed in favor of surety company not only on the building but also on the land. HELD: -The material mans lien could be charged only to the building for which the credit was made or which received the benefit of refection, the lower court was right in, holding at the interest of the mortgagee over the land is superior and cannot be made subject to the material man's lien. -Generally, real estate connotes the land and the building constructed thereon, it is obvious that the inclusion of the building in the enumeration of what may constitute real properties could only mean one thingthat a building is by itself an immovable property. -In the absence of any specific provision to the contrary, a building is an immovable property irrespective of whether or not said structure and the land on which it is adhered to belong to the same owner. -The law gives preference to unregistered refectionary credits only with respect to the real estate

upon which the refectionary or work was made. - The lien so created attaches merely to the immovable property for the construction or repair of which the obligation was incurred. Therefore, the lien in favor of appellant for the unpaid value of the lumber used in the construction of the building attaches only to said structure and to no other property of the obligors. ASSOCIATED INS. & SURETY CO., INC. V. IYA, ET AL. A building is an immovable property irrespective of where or not said structure and the land on which it is adhered to belong to the same owner. FACTS: Adriano Valino and Lucia A. Valino owns a house of strong materials. Filed a bond fr 11k subscribed by the Associated Insurance and Surety Co., Inc. and as counter-guaranty therefr, the spouses Valino executed an alleged chattel mortgage on the aforementioned house in favor of the surety company. The parcel of land on which the house is erected was still registered in the name of the Philippine Realty Corporation but was able to obtained the same from them after full payment of the purchase price. Valinos acquired another loan from Iya for 12k, executing an REM over the lot and house. However they werent able to pay off their other loan so the chattel mortgage was foreclosed. The surety company was awarded the land as the highest bidder in the auction. The surety company later on discovered that the land was subject to a REM. The surety company then requested that the house and lot be excluded from the REM. Iya, in her answer, said that she had a real right over the property and that the chattel mortgage on which the foreclosure was based should be declared null and void for non-compliance with the form required by law. CA ed only the foreclosure of the REM only up to the land and they awarded the structure to the surety company saying that the house is a personal property and may be subject to chattel mortgage. ISSUE: Which of the mortgages should have preference? RULING: Lopez v Orosa was used as a precedent here saying that the buildings an immovable itself, separate and distinct from the land. A building is an immovable property irrespective of where or not said structure and the land on which it is adhered to belong to the same owner. Only personal property is subject to a chattel mortgage and since the structure in this case is an immovable, it cannot subject to a chattel mortgage. Therefore the chattel mortgage and the sale on which it was based should be declared null and void. Iya was given the superior right not only to the land but also to the structure to foreclose them in an auction Tumalad vs Vicencio Facts: On 1 September 1955 Vicencio and Simeon,defendantsappellants, executed a chattelmortgage in favor of the Tumalads, plaintiff-appellees over their house of strong materialsover a lot in Quiapo, which were being rentedfrom Madrigal & Company, Inc. The mortgagewas executed to guarantee a loan of P4,800.00received from the Tumalads, payable within oneyear at 12% per annum. The mode of paymentwas P150.00 monthly, It was also agreed thatdefault in the payment of any of theamortizations would cause the remaining unpaid balance to become immediately due andpayable, the Chattel Mortgage enforceable, and the Sheriff of Manila authorized to sell theMortgagors property after necessarypublication. When V icencio and Simeondefaulted in paying, the mortgage wasextrajudicially foreclosed, and on 27 Mar ch1956, the house was sold at public auction pursuant to the said contract. As highest bidder, the Tumalads were issued thecorresponding certificate of sale.On 18 April 1956, the Tumalads commencedcase in the municipal court of Manila, prayingthat the house be vacated and its possessionsurrendered to them, and for Vicencio andSimeon to pay rent of P200.00 monthly up tothe time the possession is surrendered. Themunicipal court rendered its decision in favor of the Tumalads.Defendant-appellants impugned the legality of the chattel mortgage claiming that they are stillthe owner of the house but waived their rightsto introduce evidence.Nearly a year after the foreclosure sale themortgaged house had been demolished on 14and 15 January 1957 by virtue of a decisionobtained by the lessor of the land on which thehouse stood for non-payment of rentals.

Issues: 1.WON the subject matter of the mortgagewhich is a house of strong material canbe subject of real estate mortgage or achattel mortgage. 2. Whether or not the defendants arelegally bound to pay rentals to theplaintiffs during the period of 1 yearprovided by law for the redemption of the extrajudicially foreclosed house. Held: The inclusion of the building separate anddistinct from the land in the enumeration of what may c onstitute real property, that thebuilding is by itself an immovable property.However deviations hav e been allowed forvarious reasons specially if it is stipulated in thesubject of contract. In the case at bar, althoughthere is no specific statement referring to thesubject house as a personal property, yet byceding, selling or transferring a property by wayof chattel mortgage, defendantsappellantscould only have meant to convey the house asa chattel.Hence if a house belonging to a person standson a rented land belonging to another person, itmay be mortgaged as a personal property as sostipulated in the document of mortgage. Itshould be noted that the principle is predicatedon statements by the owner declaring his houseto be chattel. Party in a chattel mortgagecannot question the validity of the chattelmortgage entered into. The doctrine of estoppels therefore applies to the defendant-appellants.Since the defendant-appellants were occupyingthe house at the time the auction of sale, theyare entitled to remain in possession during theperiod of redemption or within one year fromthe date of auction sale and to collect the rentsor profits during the said period.And since the plaintiff-appellees right to posseswas not yet born at the filing of the complaint,there could be no violation or breach thereof. The Supreme Court reversed the decisionappealed from and entered another dismissingthe complaint, with costs against plaintiffs-appellees. **** Although a building is an immovable; the parties to a contract may by agreement treat as personal property that which by nature is a real property however they are estopped from subsequently claiming otherwise. **** ISSUE: Whether or not the chattel mortgage is void since its subject is an immovable HELD: NO. Although a building is by itself an immovable property, parties to a contract may treat as personal property that which by nature would be real property and it would be valid and good only insofar as the contracting parties are concerned. By principle of estoppel, the owner declaring his house to be a chattel may no longer subsequently claim otherwise. When Vicencio executed the Chattel Mortgage, it specifically provides that the mortgagor cedes, sells and transfers by way of Chattel mortgage. They intended to treat it as chattel therefore are now estopped from claiming otherwise. Also the house stood on rented land which was held in previous jurisprudence to be personalty since it was placed on the land by one who had only temporary right over the property thus it does not become immobilized by attachment. [Vicencio though was not made to pay rent since the action was instituted during the period of redemption therefore Vicencio still had a right to remain in possession of the property] Board of Assessments v Meralco Facts: The Philippine Commission enacted Act No. 484 which authorized the Municipal Board of Manila to grant a franchise to construct, maintain and operate an electric street railway and electriclight, heat and power system in the City of Manila. Meralco's electric power is generated by its hydro-electric plant located at Botocan Falls, Laguna and is transmitted to the City of Manila by means of electric transmission wires, running from the province of Laguna to the said City. These electric transmission wires which carry high voltage current, are fastened to insulators attached on steel towers constructed by respondent at intervals, from its hydro-electric plant in the province of Laguna to the City of Manila. The respondent Meralco has constructed 40 of these steel towers within Quezon City, on land belonging to it. The City Assessor of Quezon City declared the aforesaid steel towers for real property tax under Tax. Respondent paid the amount under protest, and filed a petition for review in the Court of Tax Appeals

Issue: Whether or not the Meralco poles constitutereal properties so as they can be subjected to a real property tax. Held: The SC ruled that Meralco's steel towers were considered poles within the meaning of paragraph 9 of its franchise which exempts its poles from taxation. The steel towers were considered personalty because they were removable and merely attached to square metal frames by means of bolts and could be moved from place to place when unscrewed and dismantled. Furthermore, they are not attached to an immovable in a fixed manner, and they can be separated without breaking the material or causing deterioration upon the object to which they are attached. Note: Poles - was used to denote the steel towers of an electric company engaged in the generation of hydroelectric power generated from its plant B.H. BERKENKOTTER vs .CU UNJIENG E HIJOS Short Version: MSC obtained a loan from Cu Unjieng, secured by a first mortgage on twoparcels of land and whatever forms part/is a necessary complement of the sugar mill. MSCloaned from Berkenkotter to buy additional machinery, promising that Berkenkotter would be paid as soon as MSC obtains an additional loan from Cu Unjieng; or if the loan does nottranspire, the additional machinery would be then mortgaged to Berkenkotter. This loan fromCu Unjieng did not materialize. Cu Unjieng wanted to foreclose the first mortgage, andBerkenkotter opposed this claiming that the additional machinery were now his because these were sold to him by MSC. Court held that Berkenkotter only had a right to redeem theproperty, this was the only right MSC had to transfer as the mortgage in favor of Cu Unjieng was already in force by the time MSC sold the additional property to Berkenkotter. Berkenkotters claim that the equipment are not included in the mortgage in favor of CuUnjieng as they are not real property fails. A mortgage applies even to improvements added tothe subject property subsequent to its execution. Additionally, the equipment is real innature, as defined by Art. 334(5) (now Art. 415). Facts:

Mabalacat Sugar Co., Inc. (MSC) obtained from the defendant Cu Unjieng a loansecured by a first mortgage constituted on: o Two parcels of land with all its buildings, improvements, sugar-cane mill, steelrailway, telephone line, apparatus, utensils and whatever forms part or isnecessary complement of said sugar-cane mill, steel railway, telephone line,now existing or that may in the future exist is said lots."

Shortly after said mortgage had been constituted, the MSC decided to increase thecapacity of its sugar central by buying additional machinery and equipment. o In order to carry out this plan, B.A. Green, president of MSC, proposed to theplaintiff, B.H. Berkenkotter, to advance the necessary amount for the purchaseof said machinery and equipment, promising to reimburse him as soon as hecould obtain an additional loan from the mortgagees Cu Unjieng. o B.H. Berkenkotter agreed to the proposition. MSC now purchased the additionalmachinery and equipment now in litigation.

Green then applied to Cu Unjieng for an additional loan of P75,000 offering as security the additional machinery and equipment acquired by MSC after the execution of theoriginal mortgage deed, together with whatever additional equipment acquired withsaid loan. o B.A. Green failed to obtain the loan.

As to the alleged sale of said machinery and equipment by MSC to Berkenkotter afterthey had been permanently incorporated with sugar central and while the mortgageconstituted on said sugar central to Cu Unjieng remained in force, only the right of redemption of the vendor MSC was transferred, subject to the right of Cu Unjiengunder the first mortgage. Dispositive: Petition denied. CFI affirmed DAVAO SAW MILL CO. VS. CASTILLO FACTS: Petitioner is the holder of a lumber concession. It operated a sawmill on a land, which it doesnt own. Part of the lease agreement was a stipulation in which after the lease agreement, all buildings and improvements would pass to the ownership of the lessor, which would not include machineries and accessories. In connection to this, petitioner had in its sawmill machineries and other equipment wherein some were bolted in foundations of cement. HELD: The

Some events were skipped but what seems apparent is that Cu Unjieng must have triedto enforce the mortgage agreement, including therein the equipment, after MSC soldthe equipment to Berkenkotter. o Berkenkotter contended in the CFI that the additional machinery andequipment are not permanent in character, and therefore could not besubsumed under the first mortgage deed in favor of Cu Unjieng. o The CFI ruled against Berkenkotter. o Thus this appeal by Berkenkotter from the CFI judgment ISSUE: Whether or not the additional equipment and machinery, as improvementsincorporated with the central, are subjected to the mortgage deed executed in favor of thedefendant Cu Unjieng. YES, the equipment are real in character and are thusincluded in the mortgage deed in favor of Cu Unjieng.

machinery

must

be

classified

as

personal

property.

Article 1877 of the Civil Code provides: ART. 1877. A mortgage includes all natural accessions, improvements, growing fruits,and rents not collected when the obligation falls due, and the amount of any indemnities paid or due the owner by the insurers of the mortgaged property or by virtue of the exercise of the power of eminent domain, with the declarations,amplifications, and limitations established by law, whether the estate continues in thepossession of the person who mortgaged it or whether it passes into the hands of athird person.

The lessee placed the machinery in the building erected on land belonging to another, with the understanding that the machinery was not included in the improvements which would pass to the lessor on the expiration of the lease agreement. The lessee also treated the machinery as personal property in executing chattel mortgages in favor of third persons. The machinery was levied upon by the sheriff as personalty pursuant to a writ of execution obtained without any protest being registered. Furthermore, machinery only becomes immobilized when placed in a plant by the owner of the property or plant, but not when so placed by a tenant, usufructuary, or any person having temporary right, unless such person acted as the agent of the owner. Character of Ownership Public domain Government of the Phil Islands vs Cabangis Facts: A certain lots were formerly a part of a largeparcel of land belonging to the predecessor of the herein claimants and appellees. From theyear 1896 said land began to wear away, due tothe action of the waves of Manila Bay, until theyear 1901 when the said lots becamecompletely submerged in water in ordinarytides, and remained in such a state until 1912when the Government undertook the dredgingof Vitas Estuary in order to facilitate navigation,depositing all the sand and silt taken from thebed of the estuary on the low lands which werecompletely covered with water, surroundingthat belonging to the Philippine ManufacturingCompany, thereby slowly and gradually formingthe lots, the subject matter of this proceeding. Issue Whether or not the lower court erred in not holding that the lots in question are of thepublic domain the same having been gainedfrom the sea by accession, by fillings made bythe Bureau of Public Works and by theconstruction of the break-water. Held: The Supreme Court held that the lots inquestion having disappeared on account of thegradual erosion due to the ebb and flow of thetide, and having remained in such a state untilthey were reclaimed from the sea by the fillingin done by the Government, they are publicland in the sense that neither the hereinclaimants -appellees nor their predecessors didanything to prevent their destruction.By virtue whereof, the judgment appealed fromthe lower court is reversed

Bischoff vs. Pomar and Compaia General de Tabacos And Cea vs. Villanueva :MORTGAGE OF REAL ESTATE INCLUDES IMPROVEMENTS AND FIXTURES. Itis a rule, established by the Civil Code and also by the Mortgage Law, that in amortgage of real estate, the improvements on the same are included; therefore, allobjects permanently attached to a mortgaged building or land, although they may have been placed there after the mortgage was constituted, are also included. o For machinery and other objects placed upon and used in connection with amortgaged estate to be excluded, it is indispensable that the exclusion thereof bestipulated between the contracting parties.

Article 334, paragraph 5 (now Article 415), of the Civil Code gives the character of realproperty to "machinery, liquid containers, instruments or implements intended by theowner of any building or land for use in connection with any industry or trade beingcarried on therein and which are expressly adapted to meet the requirements of suchtrade or industry. o Therefore, the machinery and equipment acquired subsequently were realproperty. o Converted to real property by reason of its purpose o Essential and principal elements of a sugar central, without them the sugarcentral would be unable to function or carry on the industrial purpose for whichit was established. o Inasmuch as the central is permanent in character, the necessary machinery and equipment installed for carrying on the sugar industry for which it has beenestablished must necessarily be permanent.

Manila International Airport Authority vs. City of Pasay Facts: Petitioner Manila International Airport Authority (MIAA) operates and administers the Ninoy Aquino International Airport (NAIA) Complex under Executive Order No. 903 (EO 903), otherwise known as the Revised Charter of the Manila International Airport Authority. Under Sections 3 and 22 of EO 903, approximately 600 hectares of land, including the runways, the airport tower, and other airport buildings, were transferred to MIAA. The NAIA Complex is located along the border between Pasay City and Paraaque City. MIAAreceived Final Notices of Real Property Tax Delinquency from the City of Pasay for the taxable years 1992 to 2001. The City of Pasay, through its City Treasurer, issued notices of levy and warrants of levy for the NAIA Pasayproperties. Thereafter, the City Mayor of Pasay threatened to sell at public auction the NAIA Pasay properties if the delinquent real property taxes remain unpaid. MIAA filed with the Court of Appeals a petition for prohibition and injunction with prayer for preliminary injunction or temporary restraining order. The petition sought to enjoin the City ofPasay from imposing real property taxes on, levying against, and auctioning for public sale the NAIA Pasay properties. Court of Appeals: Upheld the power of the City of Pasay to impose and collect realty taxes on the NAIA Pasayproperties. Sections 193 and 234 of Republic Act No. 7160 or the Local Government Code withdrew the exemption from payment of real property taxes granted to natural or juridical persons, including governmentowned or controlled corporations. Since MIAA is a government-owned corporation, it follows that its tax exemption under Section 21 of EO 903 has been withdrawn upon the effectivity of the Local Government Code. Issue: WON the NAIA Pasay properties of MIAA are exempt from real property tax YES. Held: 1. MIAA is a government "instrumentality" that does not qualify as a "government-owned or controlled corporation. Under Section 133(o) of the Local Government Code, local government units have no power to tax instrumentalities of the national government. Therefore, MIAA is exempt from any kind of tax from the local governments. A government "instrumentality" may or may not be a "government-owned or controlled corporation" (Section 2(10) of the Introductory Provisions of the Administrative Code of 1987). A government-owned or controlled corporation must be "organized as a stock or non-stock corporation." MIAA is not organized as a stock or nonstock corporation. It is not a stock corporation because it has no capital stock divided into shares. It is also not a non-stock corporation because it has no members. The Government cannot be considered as the sole member of MIAA because nonstock corporations cannot distribute any part of their income to their members. Section 11 of the MIAA Charter mandates MIAA to remit 20% of its annual gross operating income to the National Treasury. MIAA is like any other government instrumentality, but is vested with corporate powers to perform efficiently its governmental functions. When the law vests in a government instrumentality corporate powers, the instrumentality does not become a corporation. 2. The airport lands and buildings of MIAA are properties of public dominion intended for public use, and as such are exempt from real property tax under Section 234(a) of the Local Government Code. (Note: In Manila International Airport Authority v. Court of Appeals (2006MIAA rather than on the airport Authority being a taxable entity.case). In this case. Under that ruling. by law. The cited statutory definition of an "instrumentality" is incomplete. especially when we consider that MIAA exercises an essentially public function. the Court already resolved the issue of whether the airport lands and buildings of MIAAare exempt from tax under existing laws. such holding can only be for the benefit of the Republic. save for real property taxes. there is no more need to belabor the issue of whether the MIAA is a government-owned or controlled corporation (GOCC) or a government instrumentality. However. particularly on the unwieldy characterization of MIAAas a species of a government instrumentality. In this case. even if MIAA holds the record title over the airport properties. why does Section 3 of MIAA's charter authorize the President of the Philippines to approve the sale of any of these properties? While MIAA was liable for the realty taxes. The Manila International Airport Authority (MIAA) ruling was incorrectly rationalized. Otherwise. the latter could not dispose of the same through sale or through any other mode unless specifically approved by the President of the Republic. Tinga. conveyed and assigned to the ownership and administration of the MIAA shall not be disposed through sale or through any other mode unless specifically approved by the President of the Philippines. and if such. Indeed. the LGU has to find another way to collect the taxes due from MIAA. In fine. Nothing in the Local Government Code can be deemed as repealing this prohibition under Section 3. alienate. Despite the conveyance of the title to the said properties to the MIAA. then said properties are exempt from real property tax.) Opinions Ynares-Santiago. MIAA is only holding the properties for the

benefit of the Republic in its capacity as agent thereof. Nachura. Thus. MIAA properties are not of public dominion. real properties owned by the Republic of the Philippines or any of its political subdivisions are exempted from the payment of real property taxes. What needs only to be ascertained is whether the airport properties are owned by the Republic. There are no good reasons why the Court should not reassert the Mactan Cebu doctrine. Section 3 of theMIAA charter states that any portion of the lands transferred. the properties comprising the NAIA being of public dominion which pertain to the State. lease or place the airport properties as the subject matter of contracts. What need only be ascertained is whether the airport properties are owned by the Republic if the airport Authority is to be freed from the burden of paying the real property tax. The majority opinion omitted portions from Section 2(10) of the Administrative Code of 1987. dissenting. its properties could not be foreclosed upon by the local government unit seeking the taxes. The fact is that the MIAA may. The Supreme Court refuses to clarify whether its Decision in the Mactan Cebu International Airport case is deemed repealed. paving the way for a mutually acceptable negotiated solution. This strategy makes it unnecessary to determine whether MIAA is an instrumentality or a GOCC. while instrumentalities or GOCCs are generally exempted from local government taxes. Separate Opinion. it is exempt from real property tax. emphasis should be made on the ownership of the property. Thus. the same should be exempt from real property tax following Section 234(a) of the LGC. dissenting. The matter of whether MIAA is a GOCC or an instrumentality or a "government corporate entity" is irrelevant in determining whether or not the MIAA or other government instrumentalities or GOCCs are exempt from real property taxes. The court merely reiterated its ruling in that case. . Mactan Cebu provides the proper rule that. but. .The rulings in Mactan Cebu and MIAA do not really contradict. complement each one. hence. in order to determine whether airport properties are exempt from real property tax. it is Section 234. instead. MIAA then lays down the correct doctrine that airport properties are of public dominion pertaining to the state. not Section 133. of the LGC that should be determinative of the properties exempt from the said tax. falling within the ambit of Section 234(a) of the LGC. Manila International Airport Authority vs CA Facts: Manila International Airport Authority (MIAA) is the operator of the Ninoy International Airport located at Paranaque City. The Officers of Paranaque City sent notices to MIAA due to real estate tax delinquency. MIAA then settled some of the amount. When MIAA failed to settle the entire amount, the officers of Paranaque city threatened to levy and subject to auction the land and buildings of MIAA, which they did. MIAA sought for a Temporary Restraining Order from the CA but failed to do so within the 60 days reglementary period, so the petition was dismissed. MIAA then sought for the TRO with the Supreme Court a day before the public auction, MIAA was granted with the TRO but unfortunately the TRO was received by the Paranaque City officers 3 hours after the public auction. MIAA claims that although the charter provides that the title of the land and building are with MIAA still the ownership is with the Republic of the Philippines. MIAA also contends that it is an instrumentality of the government and as such exempted from real estate tax. That the land and buildings of MIAA are of public dominion therefore cannot be subjected to levy and auction sale. On the other hand, the officers of Paranaque City claim that MIAA is a government owned and controlled corporation therefore not exempted to real estate tax. Issues: Whether or not MIAA is an instrumentality of the government and not a government owned and controlled corporation and as such exempted from tax. Whether or not the land and buildings of MIAA are part of the public dominion and thus cannot be the subject of levy and auction sale. Ruling: Under the Local government code, government owned and controlled corporations are not exempted from real estate tax. MIAA is not a government owned and controlled corporation, for to become one MIAA should either be a stock or non stock corporation. MIAA is not a stock corporation for its capital is not divided into shares. It is not a non stock corporation since it has no members. MIAA is an instrumentality of the government vested with corporate powers and government functions. Under the civil code, property may either be under public dominion or private ownership. Those under public dominion are owned by the State and are utilized for public use, public service and for the development of national wealth. The ports included in the public dominion pertain either to seaports or airports. When properties under public dominion cease to be for public use and service, they form part of the patrimonial property of the State. The court

held that the land and buildings of MIAA are part of the public dominion. Since the airport is devoted for public use, for the domestic and international travel and transportation. Even if MIAA charge fees, this is for support of its operation and for regulation and does not change the character of the land and buildings of MIAA as part of the public dominion. As part of the public dominion the land and buildings of MIAA are outside the commerce of man. To subject them to levy and public auction is contrary to public policy. Unless the President issues a proclamation withdrawing the airport land and buildings from public use, these properties remain to be of public dominion and are inalienable. As long as the land and buildings are for public use the ownership is with the Republic of the Philippines REPUBLIC OF THE PHILIPPINES vs. CARLOS R. VEGA, et al., FACTS: The respondents Vegas filed an application for registration of title covering a parcel of land. They alleged that they inherited the subject land from their mother, Maria, who in turn inherited it from her father, Lorenz. Their mothers siblings died intestate, all without leaving any offspring. The Republic filed an opposition to respondents Vegas application for registration on the ground that the subject land or portions thereof were lands of the public domain and, as such, not subject to private appropriation. During the trial, respondents Vegas presented several exhibits in compliance with the jurisdictional requirements, as well as witnesses to prove respondents Vegas ownership, occupation and possession of the land subject of the registration. Significant was the testimony of Mr. Rodolfo Gonzales, a Special Investigator of the CENRO of Los Baos, Laguna, under the DENR. He attested to having conducted an inspection of the subject land and identified the corresponding Report which he had submitted to the Regional Executive Director, Region IV. The report stated that the area subject of the investigation was entirely within the alienable and disposable zone, and that there was no public land application filed for the same land by the applicant or by any other person. During the trial, respondents intervenors Buhays entered their appearance and moved to intervene in respondents Vegas application for registration. Respondents intervenors Buhays claimed a portion of the subject land consisting purportedly sold by respondents Vegasmother, Maria, to the formers predecessors in interest the sisters Gabriela and Isabel By virtue of a Bilihan ng Isang Bahagi ng Lupang Katihan. They likewise formally offered in evidence a Subdivision Plan, which indicated the portion of the subject land, which they claimed was sold to their predecessors in interest. The trial court, in its decision, granted respondents Vegas application and directed the LRA to issue the corresponding decree of registration in the name of respondents Vegas and respondents intervenors Buhays predecessors, in proportion to their claims over the subject land. The Republic appealed the Decision of the trial court, arguing that respondents Vegas failed to prove that the subject land was alienable and disposable, since the testimony of Mr.Gonzales did not contain the date when the land was declared as such. The appellate courtaffirmed in toto the decision of the trial court. ISSUE: Whether or not, based on the evidence on record, respondents Vegas have sufficiently established that the subject land is alienable and disposable? RULING: The best proofs in registration proceedings that a land is alienable and disposable are a certification from the CENRO or Provincial Environment and Natural Resources Office (PENRO) and a certified true copy of the DENRs original classification of the land. The Court, however, has nonetheless recognized and affirmed applications for land registration on other substantial and convincing evidence duly presented without any opposition from the LRA or the DENR on the ground of substantial compliance. Applying these precedents, the Court finds that despite the absence of a certification by the CENRO and a certified true copy of the original classification by the DENR Secretary, there has been substantial compliance with the requirement to show that the subject land is indeed alienable and disposable based on the evidence on record. First, respondents Vegas were able to present Mr. Gonzales of the CENRO who testified that the subject land is alienable and disposable, and who identified his written report on his inspection of the subject land. Second, Subdivision Plan Csd 04 024336, formally offered as evidence by respondents intervenors Buhays, expressly indicates that the land is alienable and disposable. Finally, upon being informed of respondents Vegas application for original registration, the LRA never raised the issue that the land subject of registration was not alienable and disposable. In

the Supplementary Report submitted during the trial court proceedings, the LRA did not interpose any objection to the application on the basis of the nature of the land VICTORIA VS. REPUBLIC This case is about the need for an applicant for registration of title to land to prove that the same has been officially declared alienable and disposable land of the public domain. The Facts and the Case petitioner Natividad Sta. Ana Victoria applied for registration under the law[1] of a lot before the Metropolitan Trial Court (MeTC) of that city. The Office of the Solicitor General (OSG), representing the respondent Republic of the Philippines, opposed the application in the usual form. Victoria testified and offered documentary evidence to show that the subject lot is a portion of a parcel of land originally owned by Victorias father Genaro Sta. Ana and previously d eclared in his name for tax purposes. Upon Genaros death, Victoria and her siblings inherited the land and divided it among themselves via a deed of partition. The Conversion/Subdivision Plan Victoria presented in evidence showed that the land is inside the alienable and disposable area as certified by the Bureau of Forest Development Victoria testified that she and her predecessors-in-interest have been in possession of the property continuously, uninterruptedly, openly, publicly, adversely and in the concept of owners since the early 1940s or for more than 30 years and have been declared as owners for taxation purposes for the last 30 years. The Republic did not present any evidence in support of its opposition. the MeTC rendered a decision,[2] granting the application for registration and finding that Victoria sufficiently established her claim and right under the land registration law to have the subject property registered in her name. the CA rendered judgment, reversing and setting aside the MeTC decision becauseVictoria failed to prove that the subject lot is alienable and disposable land of the public domain. She could not, said the CA, rely on the notation in the Conversion/Subdivision Plan she submitted before the MeTC, although it carried a notation that the land is alienable and disposable as certified by the Chief of Survey of the Land Management Services of the DENR because such notation was made only in connection with the approval of the plan. Issues Presented 1. Whether or not Victoria amply proved that the subject lot is alienable and disposable land of the public domain; and Courts Ruling Section 14(1)[4] of the Property Registration Decree has three requisites for registration of title: (a) that the property in question is alienable and disposable land of the public domain; (b) that the applicants by themselves or through their predecessors-in-interest have been in open, continuous, exclusive and notorious possession and occupation; and (c) that such possession is under a bona fide claim of ownership since June 12, 1945 or earlier.[5] A similar right is granted under Sec. 48(b) of the Public Land Act.[6] There are no material differences between Sec. 14(1) of the Property Registration Decree and Sec. 48(b) of the Public Land Act.[7] Sec. 14(1) operationalizes the registration of such lands of the public domain.[8] Here, the only reason the CA gave in reversing the decision of the MeTC is that Victoria failed to submit the November 6, 2006 Certification issued by the DENR, verifying the subject property as within the alienable and disposable land of the public domain, during the hearing before the MeTC. She belatedly submitted it on appeal. To prove that the land subject of the application for registration is alienable, an applicant must establish the existence of a positive act of the government such as a presidential proclamation or an executive order; an administrative action; investigation reports of Bureau of Lands investigators; and a legislative act or statute.[9] The applicant may secure a certification from the government that the lands applied for are alienable and disposable, but the certification must show that the DENR Secretary had approved the land classification and released the land of the pubic domain as alienable and disposable, and that the land subject of the application for registration falls within the approved area per verification through survey by the PENRO or

CENRO.[10] The applicant must also present a copy of the original classification of the land into alienable and disposable, as declared by the DENR Secretary or as proclaimed by the President.[11] In compliance, the OSG submitted a certification from the DENR stating that Senior Forest Management Specialist Corazon D. Calamno, who signed Victorias DENR Certification, is authorized to issue certifications regarding status of public land as alienable and disposable land.[14] The OSG also submitted a certified true copy of Forestry Administrative Order 4-1141 dated January 3, 1968,[15] signed by then Secretary of Agriculture and Natural Resources Arturo R. Tanco, Jr., which declared portions of the public domain covered by Bureau of Forestry Map LC2623, approved on January 3, 1968, as alienable and disposable. The CA also erred in not affirming the decision of the MeTC especially since Victoria has, contrary to the Solicitor Generals allegation, proved that she and her predecessors -in-interest had been in possession of the subject lot continuously, uninterruptedly, openly, publicly, adversely and in the concept of owners since the early 1940s. In fact, she has submitted tax declarations covering the land way back in 1948 that appeared in her fathers name. We find no reason to disturb the conclusion of the trial court that Victoria amply established her right to have the subject property registered in her name, given that she has met all the requisites for registration of title under the Property Registration Decree. WHEREFORE, the Court GRANTS the petition, REVERSES and SETS ASIDE decision of the Court of Appeals, Patrimonial property of the State Laurel v Garcia FACTS: These are two petitions for prohibition seeking to enjoin respondents, their representatives and agents from proceeding with the bidding for the sale of the 3,179 square meters of land at Ropponggi Tokyo, Japan The subject property in this case is one of the four (4) properties in Japan acquired by the Philippine government under the Reparations Agreement entered into with and is part of the indemnification to the Filipino people for their losses in life and property and their suffering during World War II. As intended, the subject property became the site of the Philippine Embassy until the latter was transferred to Nampeidai. Due to the failure of our government to provide necessary funds, the Roppongi property has remained undeveloped since that time. A proposal was presented to President Corazon C. Aquino by former Philippine Ambassador to Japan, Carlos J. Valdez, to make the property the subject of a lease agreement with a Japanese firm where, at the end of the lease period, all the three leased buildings shall be occupied and used by the Philippine government. the President issued Executive Order No. 296 entitling non-Filipino citizens or entities to avail of reparations capital goods and services in the event of sale, lease or disposition. The four properties in Japan including the Roppongi were specifically mentioned in the first Whereas clause. Amidst opposition by various sectors, the Executive branch of the government has been pushing, with great vigor, its decision to sell the reparations properties starting with the Roppongi lot. The property has twice been set for bidding at a minimum floor price at $225 million. ISSUES: (1) Can the Roppongi property and others of its kind be alienated by the Philippine Government?; and (2) Does the Chief Executive, her officers and agents, have the authority and jurisdiction, to sell the Roppongi property? In G.R. NO. 92047, apart from questioning the authority of the government to alienate the Roppongi property assails the constitutionality of Executive Order No. 296, the petitioner also questions the bidding procedures of the Committee on the Utilization or Disposition of Philippine Government Properties in Japan for being discriminatory against Filipino citizens and Filipinoowned entities by denying them the right to be informed about the bidding requirements.

HELD: The petition is granted. As property of public dominion, the Roppongi lot is outside the commerce of man. It cannot be alienated. Its ownership is a special collective ownership for general use and enjoyment, an application to the satisfaction of collective needs, and resides in the social group. The purpose is not to serve the State as a juridical person, but the citizens; it is intended for the common and public welfare and cannot be the object of appropriation. The Roppongi property is correctly classified under paragraph 2 of Article 420 of the Civil Code as property belonging to the State and intended for some public service. The fact that the Roppongi site has not been used for a long time for actual Embassy service does not automatically convert it to patrimonial property. Any such conversion happens only if the property is withdrawn from public use (Cebu Oxygen and Acetylene Co. v. Bercilles, 66 SCRA 481 [1975]). A property continues to be part of the public domain, not available for private appropriation or ownership until there is a formal declaration on the part of the government to withdraw it from being such (Ignacio v. Director of Lands, 108 Phil. 335 [1960]). An abandonment of the intention to use the Roppongi property for public service and to make it patrimonial property under Article 422 of the Civil Code must be definite. Abandonment cannot be inferred from the non-use alone specially if the non-use was attributable not to the governments own deliberate and indubitable will but to a lack of financial support to repair and improve the property (See Heirs of Felino Santiago v. Lazarao, 166 SCRA 368 [1988]). Abandonment must be a certain and positive act based on correct legal premises. A mere transfer of the Philippine Embassy to Nampeidai in 1976 is not relinquishment of the Roppongi propertys original purpose. Executive Order No. 296, though its title declares an authority to sell, does not have a provis ion in this text expressly authorizing the sale of the four properties procured from Japan for the government sector. It merely intends to make the properties available to foreigners and not to Filipinos alone in case of a sale, lease or other disposition. Rep Act No. 6657, does not authorize the Executive Department to sell the Roppongi property. It merely enumerates possible sources of future funding to augment (as and when needed) the Agrarian Reform Fund created under Executive Order No. 299. Moreover, President Aquinos approval of the recommendation by the investigating committee to sell the Roppongi property was premature or, at the very least, conditioned on a valid change in the public character of the Roppongi property. It does not have the force and effect of law since the President already lost her legislative powers. The Congress had already convened for more than a year. Assuming that the Roppongi property is no longer of public dominion, there is another obstacle to its sale by the respondents. There is no law authorizing its conveyance, and thus, the Court sees no compelling reason to tackle the constitutional issue raised by petitioner Ojeda. REPUBLIC vs. RIZALVO FACTS: Respondent Teodoro P. Rizalvo, Jr. filed before the MTC, acting as a land registration court, an application for the registration of a parcel of land. Respondent alleged that he is the owner in fee simple of the subject parcel of land, that he obtained title over the land by virtue of a Deed of Transfer and that he is currently in possession of the land. In support of his claim, he presented, among others, TaxDeclaration in his name, and Proof of Payment of real property taxes beginning in 1952 up to the time of filing of the application. Thereafter, the Office of the Solicitor General (OSG) filed an Opposition. The rendered its Decision, approving respondents application. The Republic of the Philippines through the OSG filed a Notice of Appeal. However, the CA found no merit in the appeal and promulgated the assailed Decision, affirming the trial courts decision. ISSUE: Whether or not the respondent have shown indubitably that he has complied with all the requirements showing that the property, previously part of the public domain, has become private property by virtue of his acts of possession in the manner and length of time required by law. HELD: NO. Under Section 14 (1) of PD 1529, applicants for registration of title must sufficiently establish first, that the subject land forms part of the disposable and alienable lands of the public domain;

second, that the applicant and his predecessors-in-interest have been in open, continuous, exclusive and notorious possession and occupation of thesame; and third, that it is under a bona fide claim of ownership since June 12, 1945, or earlier. The first requirement was satisfied in this case. The certification and report of the CENRO of La Union, states that the entire land areain question is within the alienable and disposable zone, certified as such since January21, 1987. Respondent has likewise met the second requirement as to ownership and possession. The MTC and the CA both agreed that respondent has presented sufficient testimonial and documentary evidence to show that he and his predecessors-in-interest were in open, continuous, exclusive and notorious possession and occupation of theland in question. However, the third requirement, that respondent and his predecessors-in-interest be in open, continuous, exclusive and notorious possession and occupation of the subject property since June 12, 1945 or earlier, has not been satisfied. Respondent only managed to present oral and documentary evidence of his and his mothers ownership and possession of the land since 1958 through a photocopy of the Deed of Absolute Sale dated July 8, 1958 between Eufrecina Navarro and Bibiana P. Rizalvo. He presented Tax Declaration for the year 1948 in the name of Eufrecina Navarro and real property tax receipts beginning in 1952. Even assuming that the 1948 Tax Declaration in the name of Eufrecina Navarro and the tax payment receipts could be taken in this case as proof of a claim of ownership, still, respondent lacks proof of occupation and possession beginning June 12, 1945 or earlier. What is categorically required by law is open, continuous, exclusive, and notorious possession and occupation under a Bonafide claim of ownership since June 12, 1945 or earlier. Indeed, even assuming arguendo that the DENR-CENRO certification and report is enough to signify that the land is no longer intended for public service or the development of the national wealth, respondent is still not entitled to Registration because the land was certified as alienable and disposable in 1987, while the application for registration was filed on December 7, 2000, a mere thirteen (13) years after and far short of the required thirty (30) years under existing laws on prescription Private ownership Viuda de Tan Toco v. Mun. Council of Iloilo FACTS: The municipality of Iloilo bought from the widow of Tan Toco a parcel of land for P42,966.40 which was used for street purposes. For failure of the municipality to pay the debt, the widow obtained a writ of execution against the municipal properties, and by virtue of such writ was able to obtain the attachment of two auto trucks used for street sprinkling, one police patrol automobile, two police stations, and two markets, including the lots on which they had been constructed. The issue is the validity of the attachment. HELD: The attachment is not proper because municipal-owned real and personal properties devoted to public or governmental purposes may not be attached and sold for the payment of a judgment against a municipality. Just as it is essential to exempt certain properties of individuals (like the bare essentials) from execution, so it is also essential and justifi able to exempt property for public use from execution, otherwise governmental service would be jeopardized. [NOTE: Had the properties been patrimonial, they could have been levied upon or attached *** the properties are public and exempt from execution. The vehicles and the police station all serve governmental functions. The market, though not purely public is also exempt because it would allow a third party to the franchise agreement to assume control w/o the approval of the administration. THE PROVINCE OF ZAMBOANGA DEL NORTE vs.CITY OF ZAMBOANGA RESOLUTION Professing respect for the principles enunciated by this Court in its decision of 28 March 1968, in Case G. R. No. L-24440, entitled Province of Zamboanga del Norte vs. City of Zamboanga, et al., 1 the appellant City seeks reconsideration of our decision in so far as the latter declares that Republic Act 3039 is unconstitutional and void in so far as the same seeks to deprive the Province of Zamboanga del Norte of its share in the 26 lots situated within the City of Zamboanga, and hereinafter enumerated, without just compensation, for the reason that said 26 lots are patrimonial property of the old Province of Zamboanga. Said 26 lots are declared in the main decision to be the following:

The movant City contends that the 26 lots aforestated were not patrimonial property of the former Province of Zamboanga, for the reason that said 26 lots have always been used for public purposes, such as school sites, playgrounds and athletic fields for schools. Appellant City of Zamboanga, therefore, prays that the main decision be partly reconsidered and that all title to, and ownership of, the 26 lots be declared to have been validly vested in said City free of charge by Republic Act No. 3039. the records are ordered remanded to the court of origin for a new trial, wherein the parties shall be given opportunity to adduce and submit any evidence in their possession to show whether or not the 26 lots aforesaid were or were not actually devoted to public use or governmental purposes prior to the enactment of Republic Act No. 3039. Thereafter, the Court of First Instance shall decide the issues anew, taking into account the evidence submitted by the parties and the principles of law laid down by this Supreme Court in its main decision of the present case, dated 28 March 1968. Zamboanga Del Norte vs. Zamboanga City (first decision) Doctrine: In the case of state properties, properties for public service are of public dominion; this is not so in the case of provinces, cities, etc., said properties for public service are patrimonial since they are not for public use. Facts: Prior to its incorporation as a chartered city, the Municipality of Zamboanga used to be the provincial capital of the then Zamboanga Province. On October 12, 1936, Commonwealth Act 39 was approved converting the Municipality of Zamboanga into Zamboanga City. Sec. 50 of the said Act also provided that Buildings and properties which the province shall abandon upon the transfer of the capital to another place will be acquired and paid for by the City of Zamboanga at a price to be fixed by the Auditor General. The properties and buildings referred to consisted of 50 lots and some buildings constructed thereon, located in the City of Zamboanga and covered individually by Torrens certificates of title in the name of Zamboanga Province. It appears that in 1945, the capital of Zamboanga Province was transferred to Dipolog. Subsequently, Republic Act 286 was approved. creating the municipality of Molave and making it the capital of Zamboanga Province. Republic Act 711 was approved dividing the province of Zamboanga into two (2): Zamboanga del Norte and Zamboanga del Sur. The Auditor General, apportioned the assets and obligations of the defunct Province of Zamboanga as follows: 54.39% for Zamboanga del Norte and 45.61% for Zamboanga del Sur. The Executive Secretary, by order of the President, issued a ruling holding that Zamboanga del Norte had a vested right as owner (should be co-owner pro-indiviso) of the properties mentioned in Sec. 50 of Commonwealth Act 39, and is entitled to the price thereof, payable by Zamboanga City. This ruling revoked the previous Cabinet Resolution conveying all the said 50 lots and buildings thereon to Zamboanga City for P1.00, effective as of 1945, when the provincial capital of the then Zamboanga Province was transferred to Dipolog. Issue: Whether all the properties concerned are patrimonial properties. Held: There are two conflicting applicable laws in the case at bar. Applying the New Civil Code, if the property is owned by the municipality (meaning municipal corporation) in its public and governmental capacity, the property is public and Congress has absolute control over it. But if the property is owned in its private or proprietary capacity, then it is patrimonial and Congress has no absolute control. The municipality cannot be deprived of it without due process and payment of just compensation. Under the said law, all the properties in question, except the two (2) lots used as High School playgrounds, could be considered as patrimonial properties of the former Zamboanga province. Even the capital site, the hospital and leprosarium sites, and the school sites will be considered patrimonial for they are not for public use. They would fall under the phrase public works for public service for it has been held that under the ejusdem generis rule, such public works must be for free and indiscriminate use by anyone, just like the preceding enumerated properties in the first paragraph of Art 424. The playgrounds, however, would fit into this category. On the other hand, applying the norm obtaining under the principles constituting the law of Municipal Corporations, all those of the 50 properties in question which are devoted to public service are deemed public; the rest remain patrimonial. Under this norm, to be considered public, it is enough that the property be held and, devoted for governmental purposes like local administration, public education, public health, etc. Under the aforementioned law, Republic Act 3039 is valid insofar as it affects the lots used as capitol site, school sites and its grounds, hospital and leprosarium sites and the high school playground sites a total of 24 lots since these were

held by the former Zamboanga province in its governmental capacity and therefore are subject to the absolute control of Congress. The records do not disclose whether they were constructed at the expense of the former Province of Zamboanga. Considering however the fact that said buildings must have been erected even before 1936 when Commonwealth Act 39 was enacted and the further fact that provinces then had no power to authorize construction of buildings such as those in the case at bar at their own expense, 14 it can be assumed that said buildings were erected by the National Government, using national funds. Hence, Congress could very well dispose of said buildings in the same manner that it did with the lots in question. But even assuming that provincial funds were used, still the buildings constitute mere accessories to the lands, which are public in nature, and so, they follow the nature of said lands, i.e., public. Moreover, said buildings, though located in the city, will not be for the exclusive use and benefit of city residents for they could be availed of also by the provincial residents. The province then and its successors-in-interest are not really deprived of the benefits thereof. But Republic Act 3039 cannot be applied to deprive Zamboanga del Norte of its share in the value of the rest of the 26 remaining lots which are patrimonial properties since they are not being utilized for distinctly, governmental purposes. The fact that these 26 lots are registered strengthens the proposition that they are truly private in nature. On the other hand, that the 24 lots used for governmental purposes are also registered is of no significance since registration cannot convert public property to private. In fine, the Court ordered herein defendant Zamboanga City to return to plaintiff Zamboanga del Norte in lump sum the amount of P43,030.11 which the former took back from the latter out of the sum of P57,373.46 previously paid to the latter. Secondly, the defendants were ordered to effect payments in favor of plaintiff of whatever balance remains of plaintiffs 54.39% sha re in the 26 patrimonial properties, after deducting therefrom the sum of P57,373.46, on the basis of Resolution No. 7 dated March 26, 1949 of the Appraisal Committee formed by the Auditor General, by way of quarterly payments from the allotments of defendant City, in the manner originally adopted by the Secretary of Finance and the Commissioner of Internal Revenue. Casimiro Development Corporation vs. Mateo FACTS: In 1988, petitioner purchased from China Bank the land in question which was previously sold by the mother of Mateo to Rodolfo Pe who in turn constituted a mortgage on the property in favor of China Bank as security for a loan. China Bank foreclosed the mortgage and consolidated its ownership of the property after Rodolfo failed to redeem. A TCT was issued in the name of China Bank. In 1991, CDC brought an action for unlawful detatiner against the respondents siblings. Respondent counters that CDC acquired the property from China Bank in bad faith because it had actual knowledge of the possession of the property by the respondent and his siblings. ISSUE: WON CDC was an innocent purchaser for value. HELD: One who deals with property registered under the Torrens system need not go beyond the certificate of title, but only has to rely on the certificate of title. He is charged with notice only of such burdens and claims as are annotated on the title. China Banks TCTs was a clean title, that is, it was free from any lien or encumbrance, CDC had the right to rely, when it purchased the property, solely upon the face of the certificate of title in the name of China Bank. The respondents siblings possession did not translate to an adverse claim of ownership. They even characterized their possession only as that of mere agricultural tenants. Under no law was possession grounded on tenancy a status that might create a defect or inflict a law in the title of the owner. CDC having paid the full and fair price of the land, was an innocent purchaser for value. The TCT in the name of CDC was declared valid and subsisting. VILMA G. ARRIOLA and ANTHONY RONALD G. ARRIOLA vs. JOHN NABOR C. ARRIOLA Facts: Fidel Arriola died and is survived by his legal heirs: John Nabor Arriola (respondent) ,his son with his first wife , and Vilma G. Arriola, his second wife and his other son, Anthony Ronald Arriola (petitioners).On Feb. 16, 2004, the RTC rendered a decision ordering the partition of the parcel of land covered by TCT No 383714 (84191)left by the decedent Fidel S. Arriola by and among his

heirs John Nabor C. Arriola, Vilma G. Arriola and Anthony Ronald G. Arriola in equal shares of onethird (1/3) each without prejudice to the rights of creditors or mortgagees thereon, if any. However, the parties failed to agree on how to divide the abovementioned property and so the respondent proposed to sell it though public auction. The petitioners initially agreed but refused to include in the auction the house standing on the subject land. The respondent then filed an Urgent Manifestation and Motion for Contempt of Court but was denied by the RTC for lack of merit. When a motion of reconsideration was still denied by the RTC, the respondent elevated the case to the CA with a petition for certiorari and prayed that he be allowed to push through with the auction of the subject land including the house built on it. The CA granted the petition and ordered the public auction sale of the subject lot including the house built on it. Petitioners filed a motion for reconsideration but the CA denied the said motion. Hence this petition for review on Certiorari Issue: The subject house is deemed part of the subject land. RULING: The right to accession is automatic (ipso jure), requiring no prior act on the part of the owner or the principal. So that even if the improvements including the house were not alleged in the complaint for partition, they are deemed included in the lot on which they stand, following the principle of accession. Consequently, the lot subject of judicial partition in this case includes the house which is permanently attached thereto, otherwise, it would be absurd to divide the principal, i.e., the lot, without dividing the house which is permanently attached thereto. BUNDLE OF RIGHTS A. Ownership Lunod et al vs. Meneses FACTS Plaintiifs (Appellees) Nicolas Lunod and 7 others are owners of farmlands on the upper estates near a lake (Calalaran). Defendant-Appellant Higno Meneses is the owner of a fishpond and a strip of land in Paraanan adjoining said lake on one side and a river on the other. Paraan is the only outlet of water to the river from the lands of Lunod et al during rainy season. In 1901 Meneses converted the land in Paraan to a fishpond and by means of a dam and a bamboo net prevented the free passage of water through Paraan causing flood and damage of plantations in the upper estates. Lunod et al filed a complaint alleging that there exists in favor of their rice fields a statutory easement for more than 20 years before 1901 and praying that Meneses be ordered to remove the obstructions that impede the passage of water through Paraanan. TC ruled in favor of the plaintiffs. ISSUE WON Meneses can be permitted to obstruct the flow of waters through his lands. HELD NO. But Lunod et al cannot prevent the defendant from building works to prevent his lands against influx of waters. RATIO Where a statutory easement exists between adjoining estates, the owner of the lower lands must not construct any work that may impair or obstruct an easement which consists in receiving the waters which naturally, and without the intervention of man, descend from more elevated lands; neither shall the owner of the latter construct any work that may increase the easement. The Civil Code allows that every owner may enclose his property by means of walls, dikes, fences, or any other device, but his right is limited by the easement with which his estate is charged.

Since the plaintiffs can not prevent the defendant from protecting his lands against the influx of salt water; but the defendant could never be permitted to obstruct the flow of the waters through his lands to the river during the heavy rains, when the high lands in Calalaran and the lake in said place are flooded, thereby impairing the right of the owners of the dominant estates; the court advised that it is perhaps useful and advantageous to all parties that Meneses be made to build a another dike in addition to the old dike between the lake of said place and the low lands in Paraanan, for the purpose of preventing the salt waters of the river flooding (at high tide) not only the lowlands in Paraanan but also the higher ones of Calalaran and its lake. ***** Meneses cannot block the flow of water. Art. 552 (A637, NCC): Lower estates must receive the waters which naturally and without intervention of man descend from the higher estates, as well as the stone or earth which they carry with them. Neither may the owner of the lower estate construct works preventing this easement, nor one of thehigher estates works increasing this burden. In addition, under the land of waters, Meneses had no right to construct the works which blocks thepassage through his land and the outlet to the river. Having done so, tothe detriment of the easement charged on his estate, he violated the law NPC V. IBRAHIM The NPC constructed underground tunnels on the property of the respondents without their knowledge and consent and without any expropriation proceeding. It contended that it constructed an easement on the property. Was there taking of the property considering that the owners were deprived of their beneficial use and enjoyment of the same, hence, entitled to just compensation? Yes. The manner in which the easement was created by the NPC, violated the due process rights of the owners as it was without notice and indemnity to them and did not go through proper expropriation proceedings. NPC could have, at any time, validly exercised the power of eminent domain to acquire the easement over the property as this power encompasses not only the taking or appropriation of title to and possession of the expropriated property but likewise covers even the imposition of a mere burden upon the owner of the condemned property. (Rep. v. PLDT, 136 Phil. 20 (1969)). Significantly, though, landowners cannot be deprived of their right over their land until expropriation proceedings are instituted in court. The court must then see to it that the taking is for pubic use, that there is payment of just compensation and that there is due process of law. In disregarding this procedure and failing to recognize the owners ownership of the sub-terrain portion, NPC took a risk and exposed itself to greater liability with the passage of time. It must be emphasized that the acquisition of the easement is not without expense.The underground tunnels imposed limitations on the owners use of the property for an indefinite period and deprived them of its ordinary use.The owners are clearly entitled to the payment of just compensation. Notwithstanding the fact that NPC only occupied the sub-terrain portion, it is liable to pay not merely an easement fee but rather the full compensation for the land. This is so because, the nature of the easement practically deprived the owners of its normal beneficial use.The owners, as the owners of the property thus expropriated, are entitled to a just compensation which should be neither more nor less, whenever it is possible to make the assessment, than the money equivalent of said property. (NPC v. Ibrahim, et al., G.R. No. 168732, June 29, 2007). ANECO REALTY vs. LANDEX THIS is a simple case of a neighbor seeking to restrain the landowner from fencing his own property. The right to fence flows from the right of ownership. Absent a clear legal and enforceable right, Will not unduly restrain the landowner from exercising an inherent proprietary right. Facts Fernandez Hermanos Development, Inc. (FHDI) is the original owner of a tract of land in San Francisco Del Monte, Quezon City. FHDI subdivided the land into thirty-nine (39) lots.[3] It later sold twenty-two (22) lots to petitioner Aneco and the remaining seventeen (17) lots to respondent Landex.[4] The dispute arose when Landex started the construction of a concrete wall on one of its lots. To restrain construction of the wall, Aneco filed a complaint for injunction[5] with the RTC in Quezon City. Aneco later filed two (2) supplemental complaints seeking to demolish the newly-built wall and to hold Landex liable for two million pesos in damages.[6] Landex filed its Answer[7] alleging, among others, that Aneco was not deprived access to its lots due to the construction of the concrete wall. Landex claimed that Aneco has its own entrance to its property along Miller Street, Resthaven Street, and San Francisco del Monte

Street. The Resthaven access, however, was rendered inaccessible when Aneco constructed a building on said street. Landex also claimed that FHDI sold ordinary lots, not subdivision lots, to Aneco based on the express stipulation in the deed of sale that FHDI was not interested in pursuing its own subdivision project. Issue: Whether or not Aneco may enjoin Landex from constructing a concrete wall on its own property. Held: The petition is without merit. Article 430 of the Civil Code gives every owner the right to enclose or fence his land or tenement by means of walls, ditches, hedges or any other means. The right to fence flows from the right of ownership. As owner of the land, Landex may fence his property subject only to the limitations and restrictions provided by law. Absent a clear legal and enforceable right, as here, No interference with the exercise of an essential attribute of ownership. Aneco failed to prove any clear legal right to prevent, much less restrain, Landex from fencing its own property. Aneco cannot rely on the road lot under the old subdivision project of FHDI because it knew at the time of the sale that it was buying ordinary lots, not subdivision lots, from FHDI. This is clear from the deed of sale between FHDI and Aneco where FHDI manifested that it was no longer interested in pursuing its own subdivision project. If Aneco wants to transform its own lots into a subdivision project, it must make its own provision for road lots. It certainly cannot piggy back on the road lot of the defunct subdivision project of FHDI to the detriment of the new owner Landex. The RTC and the CA correctly dismissed the complaint for injunction of Aneco for lack of merit. WHEREFORE, the petition is DENIED and the appealed Decision AFFIRMED. PADILLA vs.VELASCO The facts Respondents are the heirs of Dr. Artemio A. Velasco (Artemio), who died single and without any issue. During his lifetime, Artemio acquired Lot No. 2161 covered by Tax Declaration No. 4739. Artemio acquired the lot from spouses Brigido Sacluti and Melitona Obial, evidenced by a deed of sale petitioners entered the property as trustees by virtue of a deed of sale executed by the Rural Bank of Pagsanjan in favor of spouses Bartolome Solomon, Jr. and Teresita Padilla (Solomon spouses). Respondents demanded that petitioners vacate the property, but the latter refused.. respondents filed a complaint for accion publiciana, accounting and damages against petitioners before the Regional Trial Court (RTC) of Santa Cruz, Laguna. They asked the court to order petitioners to vacate the property and to pay moral and exemplary damages, attorneys fees and cost of suit. Issue: Who, as between the parties, have a better right of possession? RULING: First. The instant case is for accion publiciana, or for recovery of the right to possess. This was a plenary action filed in the regional trial court to determine the better right to possession of realty independently of the title.18Accion publiciana is also used to refer to an ejectment suit where the cause of dispossession is not among the grounds for forcible entry and unlawful detainer, or when possession has been lost for more than one year and can no longer be maintained under Rule 70 of the Rules of Court. The objective of the plaintiffs in accion publiciana is to recover possession only, not ownership.19 Based on the findings of facts of the RTC which were affirmed by the CA, respondents were able to establish lawful possession of Lot No. 2161 when the petitioners occupied the property. Lot No. 2161 based on the decision in Cadastre (Cad.). The Original Certificate of Title to the land was issued to Brigido Sacluti and Melitona Obial. On February 14, 1944, the original owners of the land sold the same to Artemio. From the date of sale, until Artemios death on January 22, 1949, he was in continuous possession of the land. When Artemio died, Isauro acted as administrator of the land with Tomas Vivero as caretaker. In 1987, petitioners occupied the property by virtue of a deed of sale between the Rural Bank of Pagsanjan and the Solomon spouses. The land bought by the Solomon spouses from the Bank is denominated as Lot No. 76-pt and previously owned by Valeriano. However, it was proved during trial that the land occupied by petitioners was Lot No. 2161 in the name of Artemio, whereas the land sold by the bank to the petitioners was Lot No. 76pt. Given this factual milieu, it can readily be deduced that respondents are legally entitled to the possession of Lot No. 2161. WHEREFORE, the instant petition is DENIED for lack of merit.

SAMPACO VS. LAMTUD FACTS: respondent Lantud, the plaintiff in the lower court, filed an action to quiet title with damages with the Regional Trial Court (RTC) against petitioner Datu Kiram Sampaco (deceased), the defendant in the lower court, who has been substituted by his heirs, represented by Hadji Soraya SampacoMacabando. Respondent alleged in his Complaint that he is the owner in fee simple of a parcel of residential lot. Petitioner Datu Kiram Sampaco, through his daughter Soraya Sampaco-Macabando with several armed men, forcibly and unlawfully entered his property and destroyedthe nursery buildings, cabbage seedlings and other improvements therein worth P10,000.00. On August30, 1984, Barangay Captain Hadji Hassan Abato and his councilmen prepared and issued a decision in writing stating that petitioner Datu Kiram Sampaco is the owner of the subject parcel of land. Respondent stated that the acts of petitioner and the said decision of the Barangay Captain may cast a cloud over or otherwise prejudice his title. Respondent stated that he and his predecessors-in-interest have been inopen, public and exclusive possession of the subject property. He prayed that the acts of petitioner andthe decision of Barangay Captain Hadji Hassan Abato and his councilmen be declared invalid, and thatpetitioner be ordered to pay respondent damages in the amount of P10,000.00 and attorneys fees. In his Answer, defendant Datu Kiram Sampaco, petitioner herein, denied the material allegations of theComplaint. Petitioner asserted that he and his predecessors-in-interest are the ones who had been in open, public, continuous, and exclusive possession of the property in dispute. Petitioner alleged that OCTNo. P-658 was secured in violation of laws and through fraud, deception and misrepresentation, considering that the subject parcel of land is a residential lot and the title issued is a free patent. Petitioner filed a counterclaim for actual and moral damages, and attorney's fees for the unfoundedcomplaint and prayed for its dismissal. He also sought the cancellation of respondents OCT No. P-658and the reconveyance of the subject parcel of land.On the other hand, petitioner Datu Kiram Sampaco testified that the land under litigation is only a portionof the 1,800 square meters of land that he inherited in 1952 from his father, Datu SampacoGubat. Since then, he had been in adverse possession and ownership of the subject lot, cultivating andplanting trees and plants through his caretaker Hadji Mustapha Macawadib. In 1962, he mortgaged theland (1,800 square meters) with the Development Bank of the Philippines, Ozamis branch. He declaredthe land (1,800 square meters) for taxation purposes and paid real estate taxes, and adduced inevidence the latest Tax Receipt No. 1756386 dated September 15, 19[9]3.RTC ruled in favor of defendant ( DATU KIRAM SAMPACO )CA reversed and ruled in favor of Hadji Serad Mingca Lantud ( Respondent )The Court of Appeals held that there is no controversy that respondent is a holder of a Torrens title;hence, he is the owner of the subject property. The appellate court stressed that Section 47 of the LandRegistration Act (Act No. 496) provides that the certificate of title covering registered land shall bereceived as evidence in all courts of the Philippines and shall be conclusive as to all matters statedtherein The Court of Appeals stated that the Torrens title has three attributes: (1) a Torrens title is the bestevidence of ownership over registered land and, unless annulled in an appropriate proceeding, the title isconclusive on the issue of ownership; (2) a Torrens title is incontrovertible and indefeasible upon theexpiration of one year from the date of the entry of the decree of registration; and (3) a Torrens title isnot subject to collateral attack. The Court of Appeals held that petitioners counterclaim filed on October 15, 1984 for cancellation of respondents original certificate of title issued on May 22, 1981 was filed beyond the statutory one-year period; hence, petitioners title had become indefeasible, and cannot be affected by the decision made byBarangay Captain Hadji Hassan Abato and his councilmen. Moreover, the appellate court held thatpetitioners prayer for the cancellation of respondents title, OCT No. P658, through a counterclaimincluded in his Answer is a collateral attack, which the law does not allow. ISSUE: 1. whether or not the Court of Appeals erred in sustaining the validity of OCT No. P-658 and confirming respondent as owner of the property in dispute. 2. Is counterclaim a collateral attack which cannot be done against the Torrens title of Respondent.

HELD: 1.NO, 2. NO.The Torrens title is conclusive evidence with respect to the ownership of the land described therein, andother matters which can be litigated and decided in land registration proceedings. Tax declarations andtax receipts cannot prevail over a certificate of title which is an incontrovertible proof of ownership. An original certificate of title issued by the Register of Deeds under an administrative proceeding is asindefeasible as a certificate of title issued under judicial proceedings. However, the Court has ruled thatindefeasibility of title does not attach to titles secured by fraud and misrepresentation. Petitioner contends in his petition that the Certification dated July 24, 1987 issued by Datu Samra I. Andam, A/Adm. Assistant II, Natural Resources District No. XII-3, Bureau of Lands, Marawi City, certifyingthat the data contained in OCT No. P-658 in respondents name had no records in the said office, showedthat respondents Torrens title was spurious.The Court holds that the certification, by itself, is insufficient to prove the alleged fraud. Fraud andmisrepresentation, as grounds for cancellation of patent and annulment of title, should never bepresumed, but must be proved by clear and convincing evidence, mere preponderance of evidence notbeing adequate. Fraud is a question of fact which must be proved. The signatory of the certification,Datu Samra Andam, A/Adm. Assistant II, Natural Resources District No. XII-3, Marawi City, was notpresented in court to testify on the due issuance of the certification, and to testify on the details of hiscertification, particularly the reason why the said office had no records of the data contained in OCT No.P-658 or to testify on the fact of fraud, if any.Thus, the Court holds that the evidence on record is insufficient to prove that fraud was committed in theissuance of respondents Torrens title. Hence, respondents Torrens title is a valid evidence of hisownership of the land in dispute.On the other hand, petitioner claims ownership of the subject lot, which is merely a portion of a larger property (1,800 square meters) that he allegedly inherited from his father in 1952, by virtue of open, publicand continuous possession of the land in the concept of owner making it petitioners private property.Hence, petitioner prays for reconveyance of the said property. Article 434 of the Civil Code governs an action for reconveyance, thus: Art. 434. In an action to recover, the property must be identified, and the plaintiff must rely on the strengthof his title and not on the weakness of the defendants claim.Under Article 434 of the Civil Code, to successfully maintain an action to recover the ownership of a realproperty, the person who claims a better right to it must prove two (2) things: first, the identity of the landclaimed; and second, his title thereto. In regard to the first requisite, in an accion reinvindicatoria, the person who claims that he has a better right to the property must first fix the identity of the land he is claiming by describing the location, area andboundaries thereof. In this case, petitioner claims that the property in dispute is part of his larger property. However, petitioner failed to identify his larger property by providing evidence of the metes and bounds thereof. Respondent has OCT No. P-658 to prove his title to the subject property, while petitioner merelyclaims that the property is already his private land by virtue of his open, public, continuouspossession of the same in the concept of owner.THE COUNTERCLAIM HERE IS A DIRECT ATTACK x x x A counterclaim can be considered a direct attack on the title. In Development Bank of the Philippinesv. Court Appeals , we ruled on the validity of a certificate of title despite the fact that the nullity thereof wasraised only as a counterclaim. It was held that a counterclaim is considered a complaint, only this time,it is the original defendant who becomes the plaintiff.The above ruling of the court on the definition of collateral attack under Section 48 of P.D. No. 1529 wasreiterated in Leyson v. Bontuyan, Heirs of Enrique Diaz v. Virata, Arangote v. Maglunob, andCatores v. Afidchao. Based on the foregoing, the Court holds that petitioners counterclaim for cancellation of respondents titleis not a collateral attack, but a direct attack on the Torrens title of petitioner. However, the counterclaimseeking for the cancellation of title and reconveyance of the subject property has prescribed as petitioner has not proven actual possession and ownership of the property due to his failure to prove the identity of his larger property that would show that the disputed property is a part thereof, and his claim of title to thesubject property by virtue of open, public and continuous possession in the concept of owner is nebulousin the light of a similar claim by respondent who holds a Torrens title to the subject property.Respondents original certificate of title was issued on May 22, 1981, while the counterclaim was filed bypetitioner on October 15, 1984, which is clearly beyond the one-year prescriptive period.In fine, the Court of Appeals did not err in confirming that respondent is the owner of the parcel of landcovered by OCT No. P-658

CORPUZ vs. Sps. AGUSTIN The Factual Antecedents Ruben C. Corpuz (Ruben) filed a complaint for ejectment against Spouses Hilarion and Justa Agustin on the allegation that he is the registered owner of two parcels of land covered by TCT issued by the Laoag City Register of Deeds Aforesaid parcels of land were formerly owned by Elias Duldulao in whose name Original Certificate of Title No. O-1717 was issued. Duldulao sold said properties on August 27, 1951 to Francisco D. Corpuz, father of Ruben C. Corpuz. The elder Corpuz allowed spouses Agustin to occupy subject properties, the latter being relatives. Despite demand to vacate, the Agustins refused to leave the premises. Ruben alleged further that he has the better right to possess subject property having acquired the same from his father, Francisco, who executed a Deed of Quitclaim in his favor The Municipal Trial Court found for the spouses Agustin and dismissed the complaint. On appeal, Branch XVI, Regional Trial Court of Laoag City affirmed said dismissal ISSUE: Who between the parties has the right to possession of the disputed properties? RULING: In the instant case, the position of respondents is that they are occupying the disputed properties as owners, having acquired these from petitioner's father through a Deed of Absolute Sale executed in 1971. Respondents believe that they cannot be dispossessed of the disputed properties, since they are the owners and are in actual possession thereof up to this date. Petitioner, however, rebuts this claim of ownership, contending that he has registered the disputed properties in his name and has been issued a land title under the Torrens system. He asserts that, having registered the properties in his name, he is the recognized owner and consequently has the better right to possession. Indeed, a title issued under the Torrens system is entitled to all the attributes of property ownership, which necessarily includes possession.17 Petitioner is correct that as a Torrens title holder over the subject properties, he is the rightful owner and is entitled to possession thereof. However, the lower courts and the appellate court consistently found that possession of the disputed properties by respondents was in the nature of ownership, and not by mere tolerance of the elder Corpuz. In fact, they have been in continuous, open and notorious possession of the property for more than 30 years up to this day. WHEREFORE, in view of the foregoing, we deny the instant Petition for lack of merit. Spouses FERNANDO and ANGELINA EDRALIN vs. PHILIPPINE VETERANS BANK Facts: Petitioners argue that Veterans Bank is not entitled to a writ of possession because it failed to properly consolidate its title over the subject property.43 They maintain that the Deed of Sale executed by the Veterans Bank in the banks own favor during the consolidation of title constitutes a pactum commissorium, which is prohibited under Article 2088 of the Civil Code.44 Respondent contends that petitioners never questioned the validity of the foreclosure proceedings or the auction sale. The failure to do so resulted in the ripening of the consolidation of ownership.45 Issue: Whether the consolidation of ownership of the extrajudicially foreclosed property through a Deed of Sale is in accordance with law RULING: Pactum commissorium is "a stipulation empowering the creditor to appropriate the thing given as guaranty for the fulfillment of the obligation in the event the obligor fails to live up to his undertakings, without further formality, such as foreclosure proceedings, and a public sale."46 "The elements of pactum commissorium, which enable the mortgagee to acquire ownership of the mortgaged property without the need of any foreclosure proceedings, are: (1) there should be a property mortgaged by way of security for the payment of the principal obligation, and (2) there should be a stipulation for automatic appropriation by the creditor of the thing mortgaged in case of non-payment of the principal obligation within the stipulated period."47

The second element is missing to characterize the Deed of Sale as a form of pactum commissorium. Veterans Bank did not, upon the petitioners default, automatically acquire or appropriate the mortgaged property for itself. On the contrary, the Veterans Bank resorted to extrajudicial foreclosure and was issued a Certificate of Sale by the sheriff as proof of its purchase of the subject property during the foreclosure sale. That Veterans Bank went through all the stages of extrajudicial foreclosure indicates that there was no pactum commissorium. WHEREFORE, premises considered, the Petition is DENIED for lack of merit. ERMITAO vs. PAGLAS Facts: respondent and petitioner, through her representative, lsabelo R. Ermitao, executed a Contract of Lease wherein petitioner leased in favor of respondent a 336 square meter residential lot and a house. The contract period is one (1) year, which commenced on November 4, 1999, with a monthly rental rate of P13,500.00. Pursuant to the contract, respondent paid petitioner P2,000.00 as security deposit to answer for unpaid rentals and damage that may be cause to the leased unit. Subsequent to the execution of the lease contract, respondent received information that sometime in March 1999, petitioner mortgaged the subject property in favor of a certain Charlie Yap (Yap) and that the same was already foreclosed with Yap as the purchaser of the disputed lot in an extra-judicial foreclosure sale which was registered on February 22, 2000. Yap's brother later offered to sell the subject property to respondent. Respondent entertained the said offer and negotiations ensued. respondent bought the subject property. A Deed of Sale of Real Property was executed by the parties as evidence of the contract. However, it was made clear in the said Deed that the property was still subject to petitioner's right of redemption. Prior to respondent's purchase of the subject property, petitioner filed a suit for the declaration of nullity of the mortgage in favor of Yap as well as the sheriff's provisional certificate of sale which was issued after the disputed house and lot were sold on foreclosure. Petitioner filed with the Municipal Trial Court in Cities (MTCC), Davao City, a case of unlawful detainer against respondent. ISSUE: who is entitled to the physical possession of the premises, that is, to the possession de facto and not to the possession de jure? RULING: Going to the main issue in the instant petition, it is settled that in unlawful detainer, one unlawfully withholds possession thereof after the expiration or termination of his right to hold possession under any contract, express or implied. In such case, the possession was originally lawful but became unlawful by the expiration or termination of the right to possess; hence the issue of rightful possession is decisive for, in such action, the defendant is in actual possession and the plaintiffs cause of action is the termination of the defendants right to continue in possession There is no dispute that at the time that respondent purchased Yap's rights over the subject property, petitioner's right of redemption as a mortgagor has not yet expired. It is settled that during the period of redemption, it cannot be said that the mortgagor is no longer the owner of the foreclosed property, since the rule up to now is that the right of a purchaser at a foreclosure sale is merely inchoate until after the period of redemption has expired without the right being exercised.16 The title to land sold under mortgage foreclosure remains in the mortgagor or his grantee until the expiration of the redemption period and conveyance by the master's deed.17 Indeed, the rule has always been that it is only upon the expiration of the redemption period, without the judgment debtor having made use of his right of redemption, that the ownership of the land sold becomes consolidated in the purchaser It, thus, clearly follows from the foregoing that, during the period of redemption, the mortgagor, being still the owner of the foreclosed property, remains entitled to the physical possession thereof subject to the purchaser's right to petition the court to give him possession and to file a bond pursuant to the provisions of Section 7 of Act No. 3135, as amended. The mere purchase and certificate of sale alone do not confer any right to the possession or beneficial use of the premises WHEREFORE, the Decision and Resolution of the Court of Appeals is AFFIRMED

Isip v Quintos Facts: PONTINO OWNS A LOT. ROGELIO SR. TOOK POSESSION OF THE LOT. PONTINO SOLD HIS LOT TO DATU BUT DATU FAILED TO PAY THE FULL PRICE. DESPITE NON-PAYMENT OF THE FULL PRICE, DATU SOLD THE LOT TO TOYO KEIKI WHO EMPLOYED ROGELIO SR. TO MANAGE ITS WATER SYSTEM PUT UP IN SAID LOT. THE LOT WAS ULTIMATELY SOLD FROM PONTINO TO DE GUZMAN. DE GUZMAN WITH RESPONDENTS PUT UP THE RONIRO ENTERPRISES WHICH TOOK THE WATER SYSTEM. ROGELIO SR. DIED BUT HIS HEIRS WERE STILL OCCUPYING THE LOT. RESPONDENTS ENTICED THE HEIRS TO PUT UP A CAR REPAIR SHOP TO UNDERTAKE REPAIRS FOR AN INSURANCE COMPANY. ON THE PRETEXT BY RESPONDENTS THAT DURING AN INSPECTION BY THE INSURANCE COMPANY THE HEIRS MUST FIRST VACATE THE PROPERTY, THE HEIRS VACATED THE PROPERTY. WHEN THEY CAME BACK THEY WERE NO LONGER ALLOWED BY RESPONDENTS TO ENTER THE PREMISES. PETITIONER FILED AN EJECTMENT CASE ON GROUND OF FORCIBLE ENTRY. PETITIONERS ARGUE THAT RESPONDENTS DEPRIVED THEM OF THE POSSESSION OF THEIR LOT THROUGH DECEIT, STRATEGY, AND STEALTH. THEY AVER THAT RESPONDENTS DECEIVED THEM TO TEMPORARILY VACATE THE PREMISES ON THE PRETEXT THAT THEY MUST CONVINCE THE INSURANCE INSPECTORS THAT THE PREMISES ARE BEING USED SOLELY FOR COMMERCIAL PURPOSES. THEY WERE THUS ALLEGEDLY TRICKED TO MOVE OUT AND ONCE THE RESPONDENTS ACHIEVED THEIR GOAL, THEY WERE PREVENTED FROM ENTERING THE PREMISES BY POSTING SECURITY GUARDS AT THE GATES. DID RESPONDENTS COMMIT FORCIBLE ENTRY? NO. THERE IS FORCIBLE ENTRY IF POSSESSION IS ILLEGAL FROM THE BEGINNING. RESPONDENTS HAVE TITLE TO THE PROPERTY. THEIR POSSESSION OF THE PROPERTY IS THEREFORE NOT ILLEGAL. THUS, THEY DID NOT COMMIT FORCIBLE ENTRY DESPITE THEIR ALLEGED STRATEGY. Under Section 1, Rule 70 of the Rules of Court, a case of forcible entry may be filed by, a person deprived of the possession of any land or building by force, intimidation, threat, strategy, or stealth x x x. In cases of forcible entry, the possession is illegal from the beginning and the basic inquiry centers on who has the prior possession de facto. XXXXXXXXXXXXXXXXXXXXXXXXXXXX PETITIONERS FURTHER ASSERT THAT THE LOT THEY OCCUPY IS DIFFERENT FROM THE LOT OCCUPIED BY THE RESPONDENTS. THEY CLAIM THAT THEIR LOT IS LOCATED AT NO. 2, BARRAMEDA ST., UPPER BICUTAN, TAGUIG WHILE THE LOT OCCUPIED BY THE RESPONDENTS IS LOCATED IN LOWER BICUTAN. CAN SC INQUIRE INTO SUCH FACT? NO. THE POINT RAISED BY THE [PETITIONERS] X X X IN RESPECT OF THE IDENTITY OF THE PROPERTY SUBJECT OF THE CONTROVERSY MAY NOT BE CONSIDERED ANYMORE AT THIS POINT SINCE IT WAS NEVER RAISED AS AN ISSUE IN THEIR APPEAL, NAY EVEN WHEN THE CASE WAS HEARD BY THE COURT A QUO. THE SUPREME COURT IS NOT A TRIER OF FACTS. ONLY QUESTIONS OF LAW MAY BE ENTERTAINED SUBJECT ONLY TO CERTAIN EXCEPTIONS, NONE OF WHICH ARE PRESENT IN THE INSTANT PETITION. IT IS THE FUNCTION OF TRIAL COURTS TO RESOLVE ACTUAL ISSUES WHOSE FINDINGS ON THESE MATTERS ARE ACCORDED RESPECT AND CONSIDERED BINDING BY THE SUPREME COURT ESPECIALLY WHEN THERE IS NO CONFLICT IN THE FACTUAL FINDINGS OF BOTH THE TRIAL COURT AND THE APPELLATE COURT. IN THIS CASE, THE METC, THE RTC AND THE CA ARE ONE IN THEIR FINDINGS THAT RESPONDENTS DID NOT FORCIBLY ENTER THE SUBJECT PREMISES. ALL THREE TRIBUNALS FOUND THAT RESPONDENTS POSSESSION IS LAWFUL AND LEGAL FROM THE BEGINNING.

VILLONDO vs. QUIJANO Factual Antecedents Respondent Carmen Quijano (Carmen) and her farm laborers, respondents Adriano Alcantara and Marcelino Ebena, intruded into her land with the help of three policemen and other barangay officials. They destroyed the plants therein, harvested the root crops, corn, and banana, built a hut, fenced off the area, and posted a "NO TRESPASSING" sign, thus preventing Valeriana and her family from entering the premises where they have always resided and depriving them of their harvest. Valeriana argued that Carmen can never assert ownership over the property because it is a government land. She claimed that Carmens parents, Rufo and Constancia Bacalla, were themselves aware that an ownership claim is worthless. Thus, they ceded their plantations on the subject land to her husband Daniel Villondo (Daniel) for P2,000.00 as declared in a "Kasabutan".5 Valeriana based her and her familys right of possession on Certificate of Stewardship No. 146099 in the name of Daniel T. Villondo,6 which she claimed to have been awarded to her nowdeceased husband whose actual name is Daniel P. Villondo. Said Certificate was issued by the Department of Environment and Natural Resources On the other hand, Carmen interposed that the alleged "Kasabutan" was never brought to her attention by her parents. In any case, she asserted that such allegation of Valeriana even supports her claim of prior possession. Carmen tacked her possessory right to that of her parents Rufo and Constancia Bacalla who in 1948 purchased11from Liberato and Vicente Abellanosa a 4.51 hectare land in Taop, Pardo, Cebu City covered by Tax Declaration No. 92638. According to her, said 4.51 hectare land includes the disputed area which her parents also cultivated and developed. Carmen submitted to the court her tax declarations over the land Issue: Whether Valeriana is a real party-in-interest in the forcible entry case she filed. RULING: Even public lands can be the subject of forcible entry cases as it has already been held that ejectment proceedings may involve all kinds of land.35 Thus, in the case at bench, while the parties are fighting over the possession of a government land, the courts below are not deprived of jurisdiction to render judgment thereon.36Courts must resolve the issue of possession even if the parties to the ejectment suit are mere informal settlers.37 For a court to restore possession, two things must be proven in a forcible entry case: prior physical possession of the property and deprivation of the property by means of force, intimidation, threat, strategy, or stealth.38"Possession de facto, [i.e., the physical possession of a property,] and not possession de jure is the only issue in a forcible entry case. This rule holds true regardless of the character of a partys possession, provided that he has in his favor priority in time. x x x"39 As used in forcible entry and unlawful detainer cases, possession refers to "physical possession, not legal possession in the sense contemplated in civil law Sans the presence of the awardee of the Certificate of Stewardship, the provision clearly allows Valeriana to institute the action for the recovery of the physical possession of the property against the alleged usurper. She has a right or interest to protect as she was the one dispossessed and thus, she can file the action for forcible entry. Any judgment rendered by the courts below in the forcible entry action will bind and definitely affect her claim to possess the subject property. The fact that Valeriana is not the holder of the Certificate of Stewardship is not in issue in a forcible entry case. This matter already delves into the character of her possession. We emphasize that in ejectment suits, it does not even matter if the partys title to the property is questionable. 45 The MTCC correctly considered Valeriana as a real party-in-interest and correctly delved strictly with the issue of physical possession. Valeriana, who has cogently convinced us that she was dispossessed of the land by force is entitled to stay on the property until she is lawfully ejected by others who can prove in a separate proceeding that they have a better right. We then end by highlighting the principle behind ejectment proceedings: xxx Regardless of the actual condition of the title to the property the party in peaceable quiet possession shall not be thrown out by a strong hand, violence or terror. Neither is the unlawful withholding of property allowed. Courts will always uphold respect for prior possession. WHEREFORE, the instant petition is hereby GRANTED

Top Management Programs Corp. v. Luis Fajardo FACTS : Emilio Gregorio filed an application for registration of title over Lots 1 to 4 of Plan Psu-204785 situated at Mag-asawang Mangga, Las Pias, Rizal, before the then Court of First Instance of Rizal; said court issued an order declaring as abandoned the reserved oppositions of Jose T. Velasquez and Pablo Velasquez. Meanwhile, Jose T. Velasquez filed an application for registration of titl eover six lots before the same court. The CFI rendered a decision declaring Gregorio to be the absolute owner of Lots 1, 2, 3 and 4described in Plan Psu-204785. On March 9, 1966, an order was issued by said court for the issuance of the decree of registration. The LRA called the attention of the Director of Lands regarding the overlapping of several lots awarded to Velasquez, with lots adjudicated to Gregorio, and requested that portions of these lots that are not in conflict be segregated. LRA informed the CFI that Lots 1 and 7 had been amended by the Bureau of Lands to exclude there from portions covered by Lot 2, Psu-64894, Psu-96904, and Lots 1 to 4, Psu-204785of Gregorio. Velasquez petitioned the CFI to set aside the award earlier made in favor of Gregorio. The CFI issued an Order declaring that the application of Velasquez be given due course insofar as Lots 1 and 7 of Ap-11135 which are identical to Lots 1 to 4, Plan Psu-204785 in favor of Gregorio respecting the same lots as null and void. Certificates of Title were issued in favor of Velasquez. Gregorio appealed the decision of the CFI to the CA. Sometime after this, he entered into an agreeent with Tomas Trinidad(Trinidad) and Luis Fajardo ( Fajardo) entitled Kasunduan na may Pambihirang Kapangyarihan. By virtue of this agreement,Fajardo would finance the cost of the litigation and in return he wouldbe entitled to one-half of the subject property after deducting twenty per cent (20%) of the total land area as attorneys fees for Trinidad if the appeal is successful. Fajardo and Trinidad filed Civil Case before the RTC of Pasig to enforce their agreement with Gregorio. The courtrendered judgment in their favour. Meanwhile, herein petitioner Top Management Programs Corporationsought the annulment of the CFI orders on the ground of extrinsic fraud. Petitioner claimed that by virtue of a notarized Deed of Absolute Sale, the heirs of Gregorio sold to it a parcel of land and a TCT covering the said property was issued in its name. The CA rendered its decision dismissing the petition for annulment. ISSUE : W/N Petitioner may quiet title and order the cancellation of the TCT in favor of Luis Fajardo. HELD : No. In an action for quieting of title, the plaintiffs must show not only that there is a cloud or contrary interest over the subject real property, but that they have a valid title to it. The court is tasked to determine the respective rights of the complainant and the other claimants, not only to place things in their proper places, and to make the claimant, who has no rights to said immovable, respect and not disturb the one so entitled, but also for the benefit of both, so thatwhoever has the right will see every cloud of doubt over the property dissipated, and he can thereafter fearlessly introduce the improvements he may desire, as well as use, and even abuse the property as he deems fit. The TCT in the name of the heirs of Emilio Gregorio, on its face showed badges of irregularity in its issuance. First, the technical description. Second, the decree number and date of issuance, as well as OCT number clearly indicate that the original decree pertained to Velasquez and not Gregorio. Third, the name of the registered owner in the original certificate is not Velasquez or Gregorio but Delta Motor Corp. And fourth, the certificate from which TCT No. 107729 was supposedly a transfer should have been the OCT (of Gregorio) and not those unfamiliar TCT numbers indicated therein. There are serious discrepancies in the registration process. In fact, it is not far-fetched that the erroneous entries could have been intended to create the impression that TCT No. 107729 was a separate and distinct title from the previously issued TCT No. S-91911 even if they pertain toone and the same lot adjudicated to Emilio Gregorio. Such conclusion is reinforced by the unexplained inaction or failure of the heirs of Gregorio to rectify the alleged errors in their title before selling the property to petitioner. The heirs of Gregorio knew that their TCT bore encumbrances in favor of third parties, notably the notice of pending litigation ( Lis Pendens) involving the property covered by said title

MANANQUIL vs. MOICO In order that an action for quieting of title may proper, it is essential that the plaintiff must have legal or equitable title to, or interest in, the property which is the subject-matter of the action. Legal title denotes registered ownership, while equitable title means beneficial ownership. In the absence of such legal or equitable title, or interest, there is no cloud to be prevented or removed. Factual Antecedents Lots 18 and 19 in Dagat-Dagatan, Navotas form part of the land previously expropriated by the National Housing Authority (NHA) and placed under its Tondo Dagat-Dagatan Foreshore Development Project where occupants, applicants or beneficiaries may purchase lots on installment basis. In October 1984, Lot 18 was awarded to spouses Iluminardo and Prescilla Mananquil under a Conditional Contract to Sell. Lot 19, on the other hand, was sold to Prescilla in February 1980 by its occupant. Iluminardo and Prescilla died without issue, but it turned out that Prescilla had a child by a previous marriage Eulogio. After the spouses death, Iluminardos supposed heirs (Mananquil heirs) his brothers and sisters and herein petitioners Dionisio and Estanislao Mananquil (Estanislao), Laudencia Mananquil-Villamor (Laudencia), and Dianita Mananquil-Rabino (Dianita) executed an Extrajudicial Settlement Among Heirs and adjudicated ownership over Lots 18 and 19 in favor of Dianita. They took possession of Lots 18 and 19 and leased them out to third parties. Sometime later, the Mananquil heirs discovered that in 1997, Eulogio and two others, Eulogio Baltazar Maypa and Brenda Luminugue, on the claim that they are surviving heirs of Iluminardo and Prescilla, had executed an Extrajudicial Settlement of Estate with Waiver of Rights and Sale, and a Deed of Absolute Sale in favor of Roberto Moico (Moico). Moico began evicting the Mananquils tenants and demolishing the structures they built on Lots 18 and 19. In June, the Mananquils instituted Civil Case No. 2741-MN for quieting of title and injunctive relief. ISSUE: THE COURT OF APPEALS ALSO COMMITTED A GRIEVOUS ERROR IN CONSTRUING THE PROVISIONS OF ARTICLES 476 AND 477 OF THE CIVIL CODE AGAINST PETITIONERS NOTWITHSTANDING THE POSITIVE CIRCUMSTANCES OBTAINING IN THIS CASE POINTING TO THE PROPRIETY OF THE CAUSE OF ACTION FOR QUIETING OF TITLE RULING: But "for an action to quiet title to prosper, two indispensable requisites must concur, namely: (1) the plaintiff or complainant has a legal or an equitable title to or interest in the real property subject of the action; and (2) the deed, claim, encumbrance, or proceeding claimed to be casting cloud on his title must be shown to be in fact invalid or inoperative despite its prima facie appearance of validity or legal efficacy."7 it appears that the petitioners have failed to show their qualifications or right to succeed Iluminardo in his rights under the NHA program/project. They failed to present any title, award, grant, document or certification from the NHA or proper government agency which would show that Iluminardo and Prescilla have become the registered owners/beneficiaries/ awardees of Lots 18 and 19, or that petitioners are qualified successors or beneficiaries under the Dagat-Dagatan program/project, taking over Iluminardos rights after his death. They did not call to the witness stand competent witnesses from the NHA who can attest to their rights as successors to or beneficiaries of Lots 18 and 19. They failed to present proof, at the very least, of the specific law, provisions, or terms that govern the Tondo Dagat-Dagatan Foreshore Development Project which would indicate a modicum of interest on their part. For this reason, their rights or interest in the property could not be established. Proof of heirship alone does not suffice; the Mananquils must prove to the satisfaction of the courts that they have a right to succeed Iluminardo under the law or terms of the NHA project, and are not disqualified by non-payment, prohibition, lack of qualifications, or otherwise. WHEREFORE, premises considered, the Petition is DENIED for lack of merit CHUNG vs. MONDRAGON, (deceased), substituted by his sisters Factual Antecedents Petitioners Joaquin G. Chung, Jr., Paz Royeras-Soler, and Mansueto Maceda are descendants of Rafael Mondragon (Rafael) by his first wife, Eleuteria Calunia (Eleuteria), while respondent Jack Daniel Mondragon5 (Jack Daniel) is Rafaels descendant by his second wife, Andrea Baldos (Andrea). Petitioners claim that from 1921 up to 2000, Rafael appeared as owner of the land in its tax declaration, and that a free patent was issued in 1987 in the name of Andreas heirs upon application of Teofila G. Maceda (Teofila), who is petitioners sister.

On the other hand, respondents claim that Andrea is the exclusive owner of the land, having inherited the same from her father Blas Baldos. They add that during Andreas lifetime, she was in lawful, peaceful and continuous possession thereof in the concept of owner petitioners filed Civil Case No. R-3248, with a prayer that Jack Daniel be declared without right to sell the land or a portion thereof; that their rights and those belonging to the legitimate heirs of Rafael and Eleuteria be declared valid and binding against the whole world; that the respondents be restrained from creating a cloud upon OCT No. 22447; and that Jack Daniels sale to Regis Schmitz be declared null and void. Issue: Wheter there is MISAPPREHENSION OF [SIC] TO THE TRUE AWARDEE OF OCT NO. 22447 TANTAMOUNT TO LACK OF JURISDICTION OVER THE CASE. RULING: The issues in a case for quieting of title are fairly simple; the plaintiff need to prove only two things, namely: "(1) the plaintiff or complainant has a legal or an equitable title to or interest in the real property subject of the action; and (2) that the deed, claim, encumbrance or proceeding claimed to be casting a cloud on his title must be shown to be in fact invalid or inoperative despite its prima facie appearance of validity or legal efficacy. Stated differently, the plaintiff must show that he has a legal or at least an equitable title over the real property in dispute, and that some deed or proceeding beclouds its validity or efficacy. It is evident from the title that the land belongs to no other than the heirs of Andrea Baldos, Rafaels second wife. The land could not have belonged to Rafael, because he is not even named in OCT No. 22447. With greater reason may it be said that the land could not belong to petitioners, who are Rafaels children by his first wife Eleuteria. Unless Eleuteria and Andrea were related by blood such fact is not borne out by the record they could not be heirs to each other. And if indeed Eleuteria and Andrea were blood relatives, then petitioners would have so revealed at the very first opportunity. Moreover, the fact that Rafael died ahead of Andrea, and that he is not even named in the title, give the impression that the land belonged solely to the heirs of Andrea, to the exclusion of Rafael. If this were not true, then the title should have as registered owners the "Heirs of Rafael and Andrea Mondragon", in which case the petitioners certainly would possess equitable title, they being descendants-heirs of Rafael. Yet OCT No. 22447 is not so written. Add to this is the fact that petitioners are not in possession of the land. A different view would have been taken if they were. Indeed, not even the fact that their sister Teofila Mac edas name appears in OCT No. 22447 could warrant a different conclusion. Her name appears therein only a representative of Andreas heirs. As mere representative, she could have no better right WHEREFORE, premises considered, the Petition is DENIED Right of Accession Bachrach vs.SEIFERT FACTS: In testate proceedings, the will of E. M. Bachrach provided for the distribution of the considerable property which he had left. The widow Mary McDonald Bachrach as administratrix and executrix had been administering the property left by her deceased husband and enjoying the usufruct thereof. The other heirs filed a petition, agreed to by usufructuary Mary McDonald Bachrach, and the Solicitor General representing the Government of the Philippines, asking that the administratrix "be authorized to pay your petitioners and until they receive their share of the estate left by the deceased E.M. Bachrach upon the death of his widow, a monthly allowance and the additional sum of P3,000 to the heir Sophie M. Seifert, who is in poor health, the said allowances to be deducted from your petitioners' share of the estate of the deceased E.M. Bachrach upon the death of the widow." Acting upon the said petition, the Court of First Instance of Manila issued an order dated October 2, 1940 granting the petition In the meantime, the administratrix Mary McDonald Bachrach, filed in the same case No. 51955 in the Court of First Instance of Manila a petition on February 19, 1947, recommending the liquidation of the assets of the estate of her deceased husband destined for charity because due to the havoc and miseries brought about by the last war, the charitable institutions to be benefited badly needed the property bequeathed to them under the will.

RULING: In the opinion of this Court, the cash in the possession of the administratrix corresponding to the of the estate adjudicated to the heirs is sufficient for the monthly allowances being paid to the heirs and that there is no necessity for the sale of the of the estate corresponding to them. The main objection to the heirs to the sale of of the estate adjudicated to them, which besides the cash already mentioned, consist mostly of shares of stock, is that said shares if sold now may not command a good price and that furthermore said heirs prefer to keep said shares intact as long as there is no real necessity for their sale. Of course, once said cash in the hands of the administratrix, corresponding to the heirs is exhausted because of the payment of the allowances made to the heirs, some other arrangements might be necessary. The administratrix would then have a right and reason to refuse the payment of said allowances from her said personal funds or from the fruits of the estate, which as a usufructuary, belong to her during her lifetime. But, until that point is reached, we see no valid reason for ordering the sale of the of the estate belonging to the heirs over their objection. In view of the foregoing, the order appealed from, insofar as it directs the sale of the one-half share of the estate destined and adjudicated to the instituted heirs, is hereby reversed. Bachrach Motors v. Talisay-Silay Milling Facts: On 22 December 1923, the Talisay-Silay Milling Co., Inc., was indebted to the PNB. To secure the payment of its debt, it succeeded in inducing its planters, among whom was Mariano Lacson Ledesma, to mortgage their land to the bank. And in order to compensate those planters for the risk they were running with their property under that mortgage, the aforesaid central, by a resolution passed on the same date, and amended on 23 March 1928, undertook to credit the owners of the plantation thus mortgaged every year with a sum equal to 2% of the debt secured according to the yearly balance, the payment of the bonus being made at once, or in part from time to time, as soon as the central became free of its obligations to the bank, and of those contracted by virtue of the contract of supervision, and had funds which might be so used, or as soon as it obtained from said bank authority to make such payment. <It seems Mariano Lacson Ledesma is indebted from Bachrach Motor; the circumstance of which is not found in the case facts.> Bachrach Motor Co., Inc. filed a complaint against the Talisay-Silay Milling Co., Inc., for the delivery of the amount of P13,850 or promissory notes or other instruments of credit for that sum payable on 30 June 1930, as bonus in favor of Mariano Lacson Ledesma. The complaint further prays that the sugar central be ordered to render an accounting of the amounts it owes Mariano Lacson Ledesma by way of bonus, dividends, or otherwise, and to pay Bachrach Motors a sum sufficient to satisfy the judgment mentioned in the complaint, and that the sale made by said Mariano Lacson Ledesma be declared null and void. The PNB filed a third party claim alleging a preferential right to receive any amount which Mariano Lacson Ledesma might be entitled from Talisay-Silay Milling as bonus. Talisay-Silay answered the complaint that Mariano Lacson Ledesmas credit (P7,500) belonged to Cesar Ledesma because he had purchase it. Cesar Ledesma claimed to be an owner by purchase in good faith. At the trial all the parties agreed to recognize and respect the sale made in favor of Cesar Ledesma of the P7,500 part of the credit in question, for which reason the trial court dismissed the complaint and cross-complaint against Cesar Ledesma authorizing the central to deliver to him the sum of P7,500. And upon conclusion of the hearing, the court held that the Bachrach Motor Co., Inc., had a preferred right to receive the amount of P11,076.02 which was Mariano Lacson Ledesmas bonus, and it ordered the central to deliver said sum to Bachrach Motors. PNB appealed. The Supreme Court affirmed the judgment appealed from, as it found no merit in the appeal;, without express finding as to costs. 1. Civil Fruits under Article 355 of the Civil Code Article 355 of the Civil Code considers three things as civil fruits: First, the rents of buildings; second, the proceeds from leases of lands; and, third, the income from perpetual or life annuities, or other similar sources of revenue. According to the context of the law, the phrase u otras analogas refers only to rents or income, for the adjectives otras and analogas agree with the noun rentas, as do also the other adjectives perpetuas and vitalicias. The civil fruits the Civil Code understands one of three and only three things, to wit: the rent of a building, the rent of land, and certain kinds of income. 2. Bonus not a civil fruit; not an income of the land The amount of the bonus, according to the resolution of the central granting it, is not based upon the value, importance or any other circumstance of the mortgaged property, but upon the total value of the debt thereby secured, according to the annual balance, which is something quite

distinct from and independent of the property referred to. As the bonus is not obtained from the land, it is not civil fruits of that land. It is neither rent of buildings, proceeds from lease of lands, or income under Article 355 of the Civil Code. OPHELIA L. TUATIS, VS. SPOUSES ELISEO ESCOL AND VISMINDA ESCOL FACTS: Tuatis and Visminda, entered into a Deed of Sale of a Part of a Registered Land by Installment in the amount of P10,000 under the following terms and conditions: 1. That the BUYER [Tuatis] shall pay to the SELLER [Visminda] the amount of THREE THOUSAND PESOS (P3,000.00), as downpayment; 2. That the BUYER [Tuatis] shall pay to the SELLER [Visminda] the amount of FOUR THOUSAND PESOS (P4,000.00), on or before December 31, 1989; 3. That the remaining balance of THREE THOUSAND PESOS (P3,000.00) shall be paid by the BUYER [Tuatis] to the SELLER [Visminda] on or before January 31, 1990; 4. That failure of the BUYER [Tuatis] to pay the remaining balance within the period of three months from the period stipulated above, then the BUYER [Tuatis] shall return the land subject of this contract to the SELLER [Visminda] and the SELLER [Visminda] [shall] likewise return all the amount paid by the BUYER [Tuatis]. Tuatis claimed that she already paid the entire purchase price and in the meantime, took possession of the subject property and constructed a residential building thereon. Tuatis requested Visminda to sign a prepared absolute deed of sale covering the subject property, but the latter refused, contending that the purchase price had not yet been fully paid. Visminda countered that, except for the P3,000.00 down payment and P1,000.00 installment paid by Tuatis on 19 December 1989 and 17 February 1990, respectively, Tuatis made no other payment to Visminda. Despite repeated verbal demands, Tuatis failed to comply with their agreement. Litigation occurs and the RTC decreed the dismissal of Tuatis' Complaint for lack of merit, the return by Tuatis of physical possession of the subject property to Visminda, and the return by Visminda of the P4,000.00 she received from Tuatis. Tuatis appeal to the CA however the appellate court dismissed the appeal for failure of Tuatis to serve and file her appellant's brief within the second extended period for the same. Visminda filed a Motion for Issuance of a Writ of Execution before the RTC on 14 January 2002. The RTC granted Visminda's Motion in a Resolution dated 21 February 2002, and issued the Writ of Execution on 7 March 2002. Tuatis thereafter filed before the RTC on 22 April 2002 a Motion to Exercise Right under Article 448 of the Civil Code of the Philippines. Tuatis moved that the RTC issue an order allowing her to buy the subject property from Visminda. While Tuatis indeed had the obligation to pay the price of the subject property, she opined that such should not be imposed if the value of the said property was considerably more than the value of the building constructed thereon by Tuatis. Tuatis alleged that the building she constructed was valued at P502,073.00, but the market value of the entire piece of land measuring 4.0144 hectares, of which the subject property measuring 300 square meters formed a part, was only about P27,000.00. Tuatis maintained that she then had the right to choose between being indemnified for the value of her residential building or buying from Visminda the parcel of land subject of the case. Tuatis stated that she was opting to exercise the second option. ISSUE: Who has the right of choice under article 448 of the civil code? HELD: The options under Article 448 are available to Visminda, as the owner of the subject property. There is no basis for Tuatis' demand that, since the value of the building she constructed is considerably higher than the subject property, she may choose between buying the subject property from Visminda and selling the building to Visminda for P502,073.00. Again, the choice of options is for Visminda, not Tuatis, to make. And, depending on Visminda's choice, Tuatis' rights as a builder under Article 448 are limited to the following: (a) under the first option, a right to retain the building and subject property until Visminda pays proper indemnity; and (b) under the second option, a right not to be obliged to pay for the price of the subject property, if it is considerably higher than the value of the building, in which case, she can only be obliged to pay reasonable rent for the same.

The rule that the choice under Article 448 of the Civil Code belongs to the owner of the land is in accord with the principle of accession, i.e., that the accessory follows the principal and not the other way around. Even as the option lies with the landowner, the grant to him, nevertheless, is preclusive. The landowner cannot refuse to exercise either option and compel instead the owner of the building to remove it from the land. VDA. DE ROXAS vs. OUR LADY'S FOUNDATION, INC. Salve Dealca Latosa filed before the RTC a Complaint for the recovery of ownership of a portion of her residential land. According to her, Atty. Henry Amado Roxas (Roxas), represented by petitioner herein, encroached on a quarter of her property by arbitrarily extending his concrete fence beyond the correct limits. After considering the evidence of all the parties, the trial court held that Latosa had established her claim of encroachment by a preponderance of evidence. It found that Roxas occupied a total of 112 square meters of Latosas lots, and that, in turn, OLFI trimmed his property by 92 square meters. Subsequently, Roxas appealed to the CA, which later denied the appeal. Since the Decision had become final, the RTC issued a Writ of Execution6 to implement the ruling ordering OLFI to reimburse Roxas for the value of the 92-square-meter property plus legal interest to be reckoned from the time the amount was paid to the third-party defendant To collect the aforementioned amount, Notices of Garnishment10 were then issued by the sheriff to the managers of the Development Bank of the Philippines and the United Coconut Planters Bank for them to garnish the account of Bishop Robert Arcilla-Maullon (Arcilla-Maullon), OLFIs general manager. Refusing to pay P1,800 per square meter to Roxas, OLFI filed a Rule 65 Petition before the CA.11 Respondent asserted that since the dispositive portion of the Decision ordered it to reimburse Roxas, it should only be made to return the purchase price that he had originally paid, which was P40 per square meter for the 92-square-meter property. Before this Court, petitioner maintains that OLFI should be made to pay P1,800, and not P40 per square meter as upheld in the 2 December 2004 Order of the RTC.13 For the immediate enforcement of the Order, petitioner further argues that because OLFI is a dummy corporation, the bank accounts of its general manager can be garnished to collect the judgment obligation of respondent.14 Issue: the determination of the correct amount to be reimbursed by OLFI to Roxas. As a corollary matter, this Court also resolves the propriety of issuing the Notices of Garnishment against the bank accounts of Arcilla-Maullon as OLFIs general manager. RULING: To settle the contention, this Court resorts to the provisions of the Civil Code governing encroachment on property. Under Article 448 pertaining to encroachments in good faith, as well as Article 450 referring to encroachments in bad faith, the owner of the land encroached upon petitioner herein has the option to require respondent builder to pay the price of the land. Although these provisions of the Civil Code do not explicitly state the reckoning period for valuing the property, Ballatan v. Court of Appeals15 already specifies that in the event that the seller elects to sell the lot, "the price must be fixed at the prevailing market value at the time of payment." More recently, Tuatis v. Spouses Escol16illustrates that the present or current fair value of the land is to be reckoned at the time that the landowner elected the choice, and not at the time that the property was purchased. We quote below the relevant portion of that Decision:17 Under the second option, Visminda may choose not to appropriate the building and, instead, oblige Tuatis to pay the present or current fair value of the land. The P10,000.00 price of the subject property, as stated in the Deed of Sale on Installment executed in November 1989, shall no longer apply, since Visminda will be obliging Tuatis to pay for the price of the land in the exercise of Vismindas rights under Article 448 of the Civil Code, and not under the said Deed. Tuatis obligation will then be statutory, and not contractual, arising o nly when Visminda has chosen her option under Article 448 of the Civil Code. Still under the second option, if the present or current value of the land, the subject property herein, turns out to be considerably more than that of the building built thereon, Tuatis cannot be obliged to pay for the subject property, but she must pay Visminda reasonable rent for the same. Visminda and Tuatis must agree on the terms of the lease; otherwise, the court will fix the terms. Therefore, we refuse to allow the execution of a corporate judgment debt against the general manager of the corporation, since in no legal sense is he the owner of the corporate property.28 Consequently, this Court sustains the CA in nullifying the Notices of Garnishment against his bank accounts.

IN VIEW THEREOF, the 25 September 2007 Decision and 11 March 2008 Resolution of the Court of Appeals in CA-GR SP No. 88622 are AFFIRMED with MODIFICATION in that the value of the 92square-meter property for which respondent should reimburse petitioner, as determined by the 2 December 2004 Order of the Regional Trial Court in Civil Case No. 5403, is hereby reinstated at P1,800 per square meter Ortiz vs. Kayanan Facts: Plaintiff used to be the legal guardian of Martin Dolorico II.When his ward died, plaintiff continued to cultivate and possess thelatters property, which was formerly a subject of homesteadapplication. In the said application, the wards uncle w as named ashis heir and successor in interest. Thus, the uncle executed anaffidavit relinquishing his rights over the property in favor of Comintan and Zamora, his grandson and son-in-law and requestedthe Director of Lands to cancel the homestead application. Thehomestead application was cancelled to the protest of Ortiz sayingthat he should be given preference to purchase the lot inasmuch as he is the actual occupant and has been in continuous possession of the same. Still, the lot in question was sold at a public auctionwherein defendant Comintan was the only bidder. The plaintiffs protest was investigated upon but his claim was notgiven due course. On appeal, respondent court rules that half of theportion of land should be given to the defendant, being thesuccessful bidder. The other half should be awarded to Zamorawithout prejudice to the right of Ortiz to participate in the publicbidding of the lot. If Ortiz is to be not declared the successful bidder, defendants should reimburse jointly said plaintiff for theimprovements introduced on the land, with him, having the right toretain the property until after he has been paid for.Plaintiff appealed the judgment. It was later found out that Ortizcollected tolls on a portion of the property wherein he has notintroduced any improvement. The judgment became final and executory. Private respondents fileda motion for its execution requesting that they file a bond in lieu of the amount that should be paid to Ortiz, on the condition that afterthe accounting of the tolls collected by plaintiff, there is still andamount due and payable to the said plaintiff, the bond shall be heldanswerable.Petitioner thus filed the instant petition, contending that in havingissued the Order and Writ of Execution, respondent Court "actedwithout or in excess of jurisdiction, and/or with grave abuse of discretion, because the said order and writ in effect vary the terms of the judgment they purportedly seek to enforce." He argued thatsince said judgment declared the petitioner a possessor in goodfaith, he is entitled to the payment of the value of the improvementsintroduced by him on the whole property, with right to retain theland until he has been fully paid such value. He likewise averred thatno payment for improvements has been made and, instead, a bondtherefor had been filed by defendants (private respondents), which,according to petitioner, is not the payment envisaged in the decisionwhich would entitle private respondents to the possession of theproperty. Furthermore, with respect to portion "B", petitioner allegesthat, under the decision, he has the right to retain the same untilafter he has participated and lost in the public bidding of the land tobe conducted by the Bureau of Lands. It is claimed that it is only inthe event that he loses in the bidding that he can be legallydispossessed thereof.It is the position of petitioner that all the fruits of the property,including the tolls collected by him from the passing vehicles, whichaccording to the trial court amounts to P25,000.00, belongs topetitioner and not to defendant/private respondent QuirinoComintan, in accordance with the decision itself, which decreed thatthe fruits of the property shall be in lieu of interest on the amount tobe paid to petitioner as reimbursement for improvements. Anycontrary opinion, in his view, would be tantamount to an amendmentof a decision which has long become final and executory and,therefore, cannot be lawfully done. The issue decisive of the controvery isafter the rendition by thetrial court of its judgment in Civil Case No. C-90 on March 22, 1966confirming the award of one-half of the property to Quirino Comintanwhether or not petitioner is still entitled to retain for his ownexclusive benefit all the fruits of the property, such as the tollscollected by him from March 1967 to December 1968, andSeptember 1969 to March 31, 1970, amounting to about P25,000.00. RULING: Negative 1. No contention that the possessor in good faith is entitled tothe fruits received before the possession is legallyinterrupted. Possession in good faith ceases or is legally interrupted from the moment defects in the title are madeknown to the possessor, by extraneous evidence or by thefiling of an action in court by the true owner for the recoveryof the property. Hence, all the fruits that the possessor mayreceive from the time he is summoned in court, or when heanswers the complaint, must be delivered and paid by him tothe owner or lawful possessor.

2. However, even after his good faith ceases, the possessor canstill retain the property (Art 546) until he has been fullyreimbursed for all the necessary and useful expenses madeby him on the property. he principal characteristic of the rightof retention is its accessory character. It is accessory to aprincipal obligation. Considering that the right of thepossessor to receive the fruits terminates when his good faithceases, it is necessary, in order that this right to retain maybe useful, to concede to the creditor the right to securereimbursement from the fruits of the property by utilizing itsproceeds for the payment of the interest as well as theprincipal of the debt while he remains in possession. 3. Petitioner cannot appropriate for his own exclusive benefitthe tolls which he collected from the property retained byhim. It was his duty under the law, after deducting thenecessary expenses for his administration, to apply suchamount collected to the payment of the interest, and thebalance to the payment of the obligation.We hold, therefore, that the disputed tolls, after deductingpetitioner's expenses for administration, belong to Quirino Comintan,owner of the land through which the toll road passed, furtherconsidering that the same was on portions of the property on whichpetitioner had not introduced any improvement. The trial court itself clarified this matter when it placed the toll road under receivership. The omission of any mention of the tolls in the decision itself may beattributed to the fact that the tolls appear to have been collectedafter the rendition of the judgment of the trial court. 4. As to the other lot, it appears that no public sale has yet beenconducted by the Bureau of Lands and, therefore, petitioneris entitled to remain in possession thereof. This is notdisputed by respondent Eleuterio Zamora. After public sale ishad and in the event that Ortiz is not declared the successfulbidder, then he should be reimbursed by respondent Zamorain the corresponding amount for the improvements on Lot5785-B Narvaez vs. Narvaez FACTS: Larry A. Ogas (Ogas) owned a 1,329-square meter parcel of land. The property was covered by Transfer Certificate of Title (TCT) No. T-1068, and a portion was subject to a 30-year lease agreement[4] with Esso Standard Eastern, Inc. Ogas sold the property to his daughter Rose O. Alciso (Alciso). TCT No. T-1068 was cancelled and TCT No. T-12422[5] was issued in the name of Alciso. Alciso entered into a Deed of Sale with Right to Repurchase,[6] selling the property to Jaime Sansano (Sansano) for P10,000. Alciso later repurchased the property from Sansano and, on 28 March 1980, she entered into another Deed of Absolute Sale,[7] this time selling the property to Celso S. Bate (Bate) for P50,000 TCT No. T-12422 was cancelled and TCT No. T-16066[9] was issued in the name of Bate. On 14 August 1981, Bate entered into a Deed of Sale of Realty,[10] selling the property to the spouses Dominador R. Narvaez and Lilia W. Narvaez (Spouses Narvaez) for P80,000. TCT No. T-16066 was cancelled and TCT No. T-16528[11] was issued in the name of the Spouses Narvaez. In 1982, the Spouses Narvaez built a commercial building on the property amounting to P300,000. Alciso demanded that a stipulation be included in the 14 August 1981 Deed of Sale of Realty allowing her to repurchase the property from the Spouses Narvaez. In compliance with Alcisos demand, the Deed stated that, The SELLER (Bate) carries over the manifested intent of the original SELLER of the property (Alciso) to buy back the same at a price under such conditions as the present BUYERS (Spouses Narvaez) may impose. The Spouses Narvaez furnished Alciso with a copy of the Deed. In a Complaint[12] dated 15 June 1984 and filed with the RTC, Alciso prayed that (1) the 25 August 1979 Deed of Sale with Right to Repurchase, the 28 March 1980 Deed of Absolute Sale, and the 14 August 1981 Deed of Sale of Realty be annulled; (2) the Register of Deeds be ordered to cancel TCT Nos. T-16066 and T-16528; (3) the Spouses Narvaez be ordered to reconvey the property; and (4) Sansano, Bate, and the Spouses Narvaez be ordered to pay damages, attorneys fees and expenses of litigation. Alciso claimed that the intention of the parties was to enter into a contract of real estate mortgage and not a contract of sale with right of repurchase. ISSUE: the Spouses Narvaez claimed that Alciso did not communicate her acceptance of the favor contained in the stipulation pour autrui; thus, she could not repurchase the property

RULING: All the requisites are present in the instant case: (1) there is a stipulation in favor of Alciso; (2) the stipulation is a part, not the whole, of the contract; (3) Bate and the Spouses Narvaez clearly and deliberately conferred a favor to Alciso; (4) the favor is unconditional and uncompensated; (5) Alciso communicated her acceptance of the favor before its revocation she demanded that a stipulation be included in the 14 August 1981 Deed of Sale of Realty allowing her to repurchase the property from the Spouses Narvaez, and she informed the Spouses Narvaez that she wanted to repurchase the property; and (6) Bate and the Spouses Narvaez did not represent, and were not authorized by, Alciso. Whether Alciso communicated to the Spouses Narvaez her acceptance of the favor contained in the stipulation pour autrui is a question of fact. It is not reviewable. The factual findings of the trial court, especially when affirmed by the Court of Appeals, are binding on the Court.[18] In its 6 April 1998 Decision, the RTC found that Alciso communicated to the Spouses Narvaez her acceptance of the favor contained in the stipulation pour autrui. Article 448 is inapplicable in cases involving contracts of sale with right of repurchase it is inapplicable when the owner of the land is the builder, sower, or planter. In Pecson v. Court of Appeals,[26]the Court held that: Article 448 does not apply to a case where the owner of the land is the builder, sower, or planter who then later loses ownership of the land by sale or donation. This Court said so in Coleongco v. Regalado: Article 361 of the old Civil Code is not applicable in this case, for Regalado constructed the house on his own land before he sold said land to Coleongco. Article 361 applies only in cases where a person constructs a building on the land of another in good or in bad faith, as the case may be. It does not apply to a case where a person constructs a building on his own land, for then there can be no question as to good or bad faith on the part of the builder. Elsewise stated, where the true owner himself is the builder of the works on his own land, the issue of good faith or bad faith is entirely irrelevant. Article 448 is inapplicable in the present case because the Spouses Narvaez built the commercial building on the land that they own. Besides, to compel them to buy the land, which they own, would be absurd. In a sale with right of repurchase, the applicable provisions are Articles 1606 and 1616 of the Civil Code, not Article 448. Articles 1606 and 1616 state: Art. 1606. The right referred to in Article 1601, in the absence of an express agreement, shall last four years from the date of the contract. Should there be an agreement, the period cannot exceed ten years. However, the vendor may still exercise the right to repurchase within thirty days from the time final judgment was rendered in a civil action on the basis that the contract was a true sale with right to repurchase. Art. 1616. The vendor cannot avail himself of the right of repurchase without returning to the vendee the price of the sale, and in addition: (1) the sale; The expenses of the contract, and any other legitimate payments made by reason of

WHEREFORE, the Court DENIES the petition. New Regent v Tanjautco Facts: The petitioner filed a complaint on rescission/declaration of nullity of contract, reconveyance and damages against the respondent. Petitioner allegedly authorized Vicente Cuevas being its Chairman and President to apply on its behalf to acquire two parcels of land by right of accretion. Cuevas applied the lot in his name and while pending approval of the application with the Bureau of Lands he assigned his rights to the respondent. An order from the Director of Lands was issued transferring rights from Cuevas to Tanjuatco. During the preliminary hearing, respondent filed a motion for demurrer of evidence after the petitioner presented their evidence. The RTC dismissed the case for insufficiencies of evidence and ruled that respondent is an innocent purchaser hence this petition for certiorari. Issue: Whether or not the court erred in dismissing the case upon demurrer of evidence? Whether or not the respondent is an innocent purchaser of the property in dispute? Ruling: As to the first issue the court held that it is a question of fact which is improper for a petition for review since the Supreme Court can only review a question of law. A question of fact exists if the doubt centers on the truth or falsity of the alleged facts. There is a question of law when the issue does not call for an examination of the probative value of evidence presented, the truth or falsehood of facts being admitted, and the doubt concerns the correct application of law and jurisprudence on the matter. The court held that to warrant a reconveyance of land where the mode of acquiring a property is by accretion, the following requisites should be met: (1) that the deposition of soil or sediment be gradual and imperceptible; (2) that it be the result of the action of the waters of the river; and (3) that the land where accretion takes place is adjacent to the banks of rivers. It is not enough to be a riparian owner in order to enjoy the benefits of accretion. One who claims the right of accretion must show by preponderant evidence that he has met all the conditions provided by law. Petitioner has notably failed in this regard as it did not offer any evidence to prove that it has satisfied the foregoing requisites. Respondent derived his title to the lands from Original Certificate of Title (OCT) No. 245 registered in the name of the Republic of the Philippines. A certification was issued confirming that said lands were verified to be Alienable and Disposable property of the State entitling it to transfer ownership to the respondent.Moreover, petitioners failed to establish fraudulent registration of ownership of the title to respondent since they did not provide evidence that Cuevas is empowered by the petitioner to apply a registration of the property in their behalf. The respondent may safely rely on what appears on the face of the registered title hence he is a buyer in good faith. Petitioner was not able to substantiate its claim for ownership of the propertytherefore their claim for reconveyance should be denied. FERNANDO vs. ACUNA At the heart of this controversy is a parcel of land covered by Original Certificate of Title (OCT) registered in the names of Jose A. Fernando, married to Lucila Tinio, and Antonia A. Fernando, married to Felipe Galvez. When they died intestate, the property remained undivided. Petitioners herein namely, Jose Fernando, Jr., Zoilo Fernando, Norma Fernando Banares, Rosario Fernando Tangkencgo, the heirs of Tomas Fernando, the heirs of Guillermo Fernando, the heirs of Iluminada Fernando and the heirs of Germogena Fernando are the heirs and successors-in-interest of the deceased registered owners. However, petitioners failed to agree on the division of the subject property amongst themselves, even after compulsory conciliation before the Barangay Lupon. Thus, petitioners, except for the heirs of Germogena Fernando, filed a Complaint4 for partition against the heirs of Germogena Fernando In his Complaint in Intervention respondent Leon Acuna (Acuna) averred that in the Decision7 dated November 29, 1929 of the Cadastral Court of Baliuag, Bulacan, the portion of the property identified as Lot 1303 was already adjudicated to: (a) Antonio Fernando, married to Felisa Camacho; (b) spouses Jose Martinez and Gregoria Sison; (c) spouses Ignacio de la Cruz and Salud Wisco; and (d) Jose Fernando, married to Lucila Tinio, the petitioners predecessor -in-

(2) The necessary and useful expenses made on the thing sold. Under Article 1616, Alciso may exercise her right of redemption by paying the Spouses Narvaez (1) the price of the sale, (2) the expenses of the contract, (3) legitimate payments made by reason of the sale, and (4) the necessary and useful expenses made on the thing sold. In the present case, the cost of the building constitutes a useful expense. Useful expenses include improvements which augment the value of the land

interest. He likewise claimed that in a 1930 Decision of the Cadastral Court, the portion identified as Lot 1302 was also already adjudicated to other people as well. ISSUE: Whether or not the ownership of Lot 1303 and the Sapang Bayan portion of the piece of land covered by O.C.T. No. RO-487 (997) or Plan Psu-39080 should revert to the descendants and heirs of the late spouses Jose Fernando and Lucila Tinio and Antonia Fernando, married to Felipe Galvez RULING: As for the issue of the ownership of Sapang Bayan, we sustain the appellate court insofar as it ruled that petitioners failed to substantiate their ownership over said area. However, we find that the Court of Appeals erred in ruling that the principle of accretion is applicable. The said principle is embodied in Article 457 of the Civil Code which states that "[t]o the owners of lands adjoining the banks of rivers belong the accretion which they gradually receive from the effects of the current of the waters." We have held that for Article 457 to apply the following requisites must concur: (1) that the deposit be gradual and imperceptible; (2) that it be made through the effects of the current of the water; and (3) that the land where accretion takes place is adjacent to the banks of rivers.45The character of the Sapang Bayan property was not shown to be of the nature that is being referred to in the provision which is an accretion known as alluvion as no evidence had been presented to support this assertion. Furthermore, in Celestial v. Cachopero,50 we similarly ruled that a dried-up creek bed is property of public dominion: A creek, like the Salunayan Creek, is a recess or arm extending from a river and participating in the ebb and flow of the sea. As such, under Articles 420(1) and 502(1) of the Civil Code, the Salunayan Creek, including its natural bed, is property of the public domain which is not susceptible to private appropriation and acquisitive prescription. And, absent any declaration by the government, that a portion of the creek has dried-up does not, by itself, alter its inalienable character. WHEREFORE, premises considered, the petition is hereby DENIED REPUBLIC vs.SANTOS By law, accretion - the gradual and imperceptible deposit made through the effects of the current of the water- belongs to the owner of the land adjacent to the banks of rivers where it forms. The drying up of the river is not accretion. Hence, the dried-up river bed belongs to the State as property of public dominion, not to the riparian owner, unless a law vests the ownership in some other person. FACTS: Alleging continuous and adverse possession of more than ten years, respondent Arcadio Ivan A. Santos III (Arcadio Ivan) applied on March 7, 1997 for the registration of Lot 4998-B (the property) in the Regional Trial Court (RTC) in Parafiaque City. The property, which had an area of 1,045 square meters, more or less, was located in Barangay San Dionisio, Paraaque City, and was bounded in the Northeast by Lot 4079 belonging to respondent Arcadio C. Santos, Jr. (Arcadio, Jr.), in the Southeast by the Paraaque River, in the Southwest by an abandoned road, and in the Northwest by Lot 4998-A also owned by Arcadio Ivan.1 Arcadio Ivan amended his application for land registration to include Arcadio, Jr. as his coapplicant because of the latters co-ownership of the property. He alleged that the property had been formed through accretion and had been in their joint open, notorious, public, continuous and adverse possession for more than 30 years.2 The City of Paraaque (the City) opposed the application for land registration, stating that it needed the property for its flood control program; that the property was within the legal easement of 20 meters from the river bank; and that assuming that the property was not covered by the legal easement, title to the property could not be registered in favor of the applicants for the reason that the property was an orchard that had dried up and had not resulted from accretion ISSUE: Whether or not Article 457 of the Civil Code was applicable RULING: The CA grossly erred in applying Article 457 of the Civil Code to respondents benefit Article 457 of the Civil Code provides that "(t)o the owners of lands adjoining the banks of rivers belong the accretion which they gradually receive from the effects of the currents of the waters."

Accretion is the process whereby the soil is deposited along the banks of rivers.12 The deposit of soil, to be considered accretion, must be: (a) gradual and imperceptible; (b) made through the effects of the current of the water; and (c) taking place on land adjacent to the banks of rivers.13 Accordingly, respondents should establish the concurrence of the elements of accretion to warrant the grant of their application for land registration. However, respondents did not discharge their burden of proof. They did not show that the gradual and imperceptible deposition of soil through the effects of the current of the river had formed Lot 4998-B. Instead, their evidence revealed that the property was the dried-up river bed of the Paraaque River, leading both the RTC and the CA to themselves hold that Lot 4998-B was "the land which was previously part of the Paraaque River xxx (and) became an orchard after it dried up." The RTC and the CA grossly erred in treating the dried-up river bed as an accretion that became respondents property pursuant to Article 457 of the Civil Code. That land was definitely not an accretion. The process of drying up of a river to form dry land involved the recession of the water level from the river banks, and the dried-up land did not equate to accretion, which was the gradual and imperceptible deposition of soil on the river banks through the effects of the current. In accretion, the water level did not recede and was more or less maintained. Hence, respondents as the riparian owners had no legal right to claim ownership of Lot 4998-B. Considering that the clear and categorical language of Article 457 of the Civil Code has confined the provision only to accretion, we should apply the provision as its clear and categorical language tells us to. Axiomatic it is, indeed, that where the language of the law is clear and categorical, there is no room for interpretation; there is only room for application.16 The first and fundamental duty of courts is then to apply the law.17 The State exclusively owned Lot 4998-B and may not be divested of its right of ownership. Article 502 of the Civil Code expressly declares that rivers and their natural beds are public dominion of the State.18 It follows that the river beds that dry up, like Lot 4998-B, continue to belong to the State as its property of public dominion, unless there is an express law that provides that the dried-up river beds should belong to some other person Co-Ownership PARDELL vs. BARTOLOME CASE DOCTRINES Hereditary succession gives rise to co-ownership. Co-ownership; extent "Each co-owner may use the things owned in common, provided he uses them in accordance with their object andin such manner as not to injure the interests of the community nor prevent the coowners from utilizing themaccording to their rights." (Article 394 of the Civil Code, now Art. 486) Matilde Ortiz and her husband occupied the upper story, designed for use as a dwelling, in the house of joint ownership; but the record shows no proof that, by so doing, the said Matilde occasioned any detriment tothe interests of the community property, nor that she prevented her sister Vicenta from utilizing the said upperstory according to her rights. It is to be noted that the stores of the lower floor were rented and an accountingof the rents was duly made to the plaintiffs.Each co-owner of realty held pro indiviso exercises his rights over the whole property and may use andenjoy the same with no other limitation than that he shall not injure the interests of his coowners, for thereason that, until a division be made, the respective part of each holder can not be determined and every one of the coowners exercises together with his other coparticipants, joint ownership over the pro indiviso property, inaddition to his use and enjoyment of the same. Co-owner not required to pay rent upon using the co-owned property; stranger to pay rent Gaspar de Bartolome, occupied for four years a room or a part of the lower floor of the same house onCalle Escolta, using it as an office for the justice of the peace, a position which he held in the capital of thatprovince, strict justice requires that he pay his sister-in-law, the plaintiff, one-half of the monthly rent which thesaid quarters could have produced, had they been leased to another person. Xxx even as the husband of thedefendant coowner of the property, he had no right to occupy and use gratuitously the said part of the lower floor of the house in question, where he lived with his wife, to the detriment of the plaintiff Vicenta who did notreceive one-half of the rent which those quarters could and should have produced, had they been occupied by astranger, in the same manner that rent was obtained from the rooms on the lower floor that were used as stores

Jose S. Dailisan vs. CA FACTS: This is a Petition for Review of theDecision and Resolution of the CA which reversed and setaside the Decision of the RTC-QC. Petitioner filed a Complaint for partition before the RTC-QC, alleging that he purchased 1/4 of the land of Federico Pugao identified as Lot 16 in Bago Bantay, QCcovered by a TCT. According to petitioner, he had paid Federico severalinstallments which totaled to P6K. When the mortgagewas released, petitioner demanded the execution of adeed of absolute sale. Instead, Federico proposed tomortgage the property to petitioner as security for a P10K loan, payable in 3 months, and upon payment of the loanthe deed of absolute sale would be executed. When Petitioner asked for the partition of the lot Federico refused and even sent a notice of evictionagainst petitioner. According to Federico, he allowed petitioner and hisniece to occupy one 1/4 of his lot, and admitted that heexecuted in favor of petitioner a deed of real estatemortgage. The loan was paid, mortgage cancelled. Federico alleged that petitioner made him sign pages of what the former told him to be parts of the real estatemortgage. He filed a complaint for falsification andejectment against petitioner. He passed away while thiscase was pending before the trial court. RTC found that respondentsfailed to disprove thevalidity of the deed of absolute sale, ruled in favor of petitioner and ordered the partition of the subjectproperty. MR was denied. On appeal, CA granted and noted that petitioner shouldhave filed an action for specific performance to compelFederico to honor the deed of absolute sale; but hadalready expired. CA noted that petitioner " filed the instant action for partition simply because it is not barred by prescription. " It ruled that the sale was void because theres no consent and that there was no proof of paymentof the price or consideration. MR was denied. ISSUES: Was deed of absolute sale valid? HELD: The notarized deed of absolute sale as a publicdocument has in its favor the presumption of regularity whichmay only be rebutted by evidence so clear, strong andconvincing as to exclude all controversy as to the falsity of thecertificate. The burden of proof to overcome the presumptionlies on the party contesting such execution. The action or defense for the declaration of the inexistence of a contract does not prescribe. Respondents' claim is that the deed of sale is a voidable, andnot void, contract because of mistake and/or fraud.Respondents stress Federico's low educational attainment andinability to understand the English language.Nevertheless, one of the respondents testified that she wasaware of the existence of the deed of sale way back in 1984. Yet, none of them took any action to annul the deed within theprescribed 4-year period which expired in 1988. According to Art. 1332, it would have been incumbent uponpetitioner to show that he fully explained the terms of thecontract to Federico if not for a crucial point. Respondentshave lost both their right to file an action for annulment or toset up such nullity of the deed of sale as a defense in an actionto enforce the same.Respondents were unable to overcome the presumption of validity of the deed of absolute sale as well as the regularity inits execution.CA ruled that petitioner's cause of action has prescribedfollowing its conclusion that petitioner's action is actually onefor specific performance, not partition. Interestingly,petitioner suddenly changed tack and declared that his originalaction was indeed an action for specific performance. He should not have gone that far. His objective is to make Federico honor their contract and perform his obligation todeliver a separate title covering the lot he sold to him butwhich can be done only after the portion is segregated from the rest of Federico's property.Petitioner's action before the RTC was properly captioned asone for partition because there are sufficient allegations in thecomplaint that he is a co-owner of the property. The regime of CO-OWNERSHIP exists when ownership of an undivided thing or right belongs to

different persons. By the nature of a co-ownership, a co-owner cannot point to a specific portion of the property owned in common as his own because his share therein remains intangible. The description "undivided 1/4 portion" shows that the portionsold is still undivided and not sufficiently identified. While thedescription provides a guide for identifying the location of thelot sold, there was no indication of its exact metes and bounds.This is the reason why petitioner was constrained to cause thesurvey of the property. As a co-owner of the property,therefore, petitioner has the right to demand partition, aright which does not prescribe. Ownership of the thing sold is acquired only from the time of delivery, either actual or constructive. Article 1498 provides that when the sale is made through a public instrument, the execution shall be equivalent to the delivery of the thingwhich is the object of the contract, if from the deed thecontrary does not appear or cannot be inferred. The Court notes that Federico had already delivered the portion he sold to petitioner , subject to the execution of a technical survey,when he executed the deed of absolute sale, which is a publicinstrument. In view of the delivery in law, coupled with petitioner's actualoccupation of the portion where his house stands, all that isneeded is its segregation from the rest of the property. WHEREFORE, the petition is GRANTED. The challenged Decision and Resolution are SET ASIDE, and the Decision of the RTC-QC is REINSTATED CRUZ v. CATAPANG, Co-owners cannot devote common property to his or her exclusive use to the prejudice of the coownership. Petitioners Leonor Cruz, Luz Cruz and Norma Maligaya are the co-owners of aparcel of land covering an area of 1,435 square meters located at Barangay MahabangLudlod, Taal, Batangas. Sometime in 1992, Teofila Catapang, with the consent of NormaMaligaya as one of the aforementioned co-owners, built a house on a lot adjacent to thesubject parcel of land. The house built by Catapang intruded on a portion of the co-owned property.In September 1995, Cruz learned about the intrusion and made several demandsfor Catapang to demolish and vacate the part of the structure encroaching upon theirproperty. However, Catapang refused and disregarded the demands of Cruz.Cruz then filed a complaint for forcible entry against Catapang before the MCTCof Taal, Batangas. The MCTC decided in favor of Cruz, ruling that consent of only oneof the co-owners is not sufficient t o justify defendants construction of the house and possession of the portion of the lot in question. On appeal, the RTC affirmed thedecision of the MCTC.Catapang filed a petition for review with the Court of Appeals, which reversed the RTCs decision and ruled in favor of her. The Court of Appeals held that there is no cause of action for forcible entry in this case because respondents entry into the property, considering the consent given by co-owner Norma Maligaya, cannot becharacterized as one made through strategy or stealth which gives rise to a cause ofaction for forcible entry. Thus, the case went to the Supreme Court. ISSUE: 1. Whether the consent given by one of the co-owners is sufficient to warrant thedismissal of a complaint for forcible entry DECISION: No, Co-owners cannot devote common property to his or her exclusive use to theprejudice of the co-ownership. In this case, the act of Norma Maligaya is tantamount todevoting the property to her exclusive use. Under Article 491 of the Civil Code, none ofthe co-owners shall, without the consent of the others, make alterations in the thingowned in common. The Court ruled that it would necessarily follow that none of the co-owners can, without the consent of the other coowners, validly give consent to themaking of an alteration by another person, such as Catapang in this case, in the thingowned in common.In addition, Article 486 of the same Code states each coowner may use the thingowned in common provided he does so in accordance with the purpose for which it isintended and in such a way as not to injure the interest of the co-ownership or preventthe other co-owners from using it according to their rights. The Court ruled that, to giveconsent to a third person to construct a house on the co-owned property would be toinjure the interest of the co-ownership and would prevent other co-owners from usingthe property in accordance with their rights.In this case, the consent of only one co-owner will not warrant the dismissal ofthe complaint for forcible entry filed against the respondent Catapang. The consentgiven by Norma Maligaya in the absence of the consent of her other coowners did notgrant Catapang any right to enter and even build upon the co-owned property.According to the Supreme Court, the respondent Catapang

s act of getting only the consent of one co-owner, her sister Norma Maligaya, and allowing the latter to stay inthe constructed house, can in fact be considered as a strategy which she utilized in orderto enter into the co-owned property. As such, respondents acts constitute forcible entry. The petition was GRANTED Plasabas v CA recovery of title implead Facts: In 1974, petitioners filed a complaint for recovery of title. Respondents claim that the subject land was inherited by all the parties from their common ancestor, Francisco Plasabas.6 Revealed in the course of the trial was that petitioner Nieves, contrary to her allegations in the complaint, was not the sole and absolute owner of the land. What the Court wants to say here is that the instant case should have been dismissed without prejudice a long time ago for lack of cause of action as the plaintiffs spouses Marcos Malazarte and Nieves Plasabas Malazarte have no complete legal personality to sue by themselves alone without joining the brothers and sisters of Nieves The appellate court affirmed the ruling of the trial court. The CA, further, declared that the nonjoinder of the indispensable parties would violate the principle of due process, and that Article 487 of the Civil Code could not be applied considering that the complaint was not for ejectment, but for recovery of title or a reivindicatory action. Ratio: Article 487 of the Civil Code provides that any one of the co-owners may bring an action for ejectment. The article covers all kinds of actions for the recovery of possession, including an accion publiciana and a reivindicatory action. A co-owner may file suit without necessarily joining all the other co-owners as co-plaintiffs because the suit is deemed to be instituted for the benefit of all. Any judgment of the court in favor of the plaintiff will benefit the other co-owners, but if the judgment is adverse, the same cannot prejudice the rights of the unimpleaded co-owners. Thus, petitioners, in their complaint, do not have to implead their co-owners as parties. The only exception to this rule is when the action is for the benefit of the plaintiff alone who claims to be the sole owner and is, thus, entitled to the possession thereof. In such a case, the action will not prosper unless the plaintiff impleads the other co-owners who are indispensable parties. The property is co-owned by Nieves and her siblings, and that petitioners have been authorized by the co-owners to pursue the case on the latters behalf. Even if a co-owner sells the whole property, the sale will affect only his own share and not the share of the other co-owners who didn't consent to the sale. Since this is the case, a co-owner is entitled to sell his undivided share, a sale of the entire property by one co-owner without the consent of the others is not null and void. REPUBLIC VS. HEIRS OF FRANCISCA DIGNOS-SORONO FACTS: 2 were adjudicated by the then Court of First Instance equal shares: a) Francisca Dignos, married to Blas Sorono b) Tito Dignos share in the two lots; c) predecessors-in-interest of the respondents and d) predecessors-in-interest of the respondents of Cebu in favor of the following in four share in the two lots; share in the two lots; share in the two lots

their shares in the lots of which they have been in continuous peaceful possession. Respondents furthermore alleged that neither petitioner nor its predecessor-in-interest had given them any written notice of its acquisition of the share of Tito Dignos. The Republic, represented by the MCIAA in its Answer with Counterclaim, maintained that from the time the lots were sold to its predecessor-in-interest CAA, it has been in open, continuous, exclusive, and notorious possession thereof; through acquisitive prescription, it had acquired valid title to the lots since it was a purchaser in good faith and for value; and assuming arguendo that it did not have just title, it had, by possession for over 30 years, acquired ownership thereof by extraordinary prescription. At all events, petitioner contended that respondents action was barred by estoppel and laches. The trial court found for respondents. the CA affirmed the trial courts decision. Hence, the present petition for review on certiorari ISSUE: 1. WON the sale of the entire 2 lots by the heirs of Tito binding to the respondents 2. WON estoppel and laches should work against respondents HELD: the petition is denied 1. NO. Article 493 of the Civil Code provides: Each co-owner shall have the full ownership of his part and of the fruits and benefits pertaining thereto, and he may therefore alienate, assign or mortgage it, and even substitute another person in its enjoyment, except when personal rights are involved. But the effect of the alienation of the mortgage, with respect to the co-owners, shall be limited to the portion which may be allotted to him in the division upon the termination of the co-ownership. Apropos is the following pertinent portion of this Courts decision in Bailon-Casilao v. CA: As early as 1923, this Court has ruled that even if a co-owner sells the whole property as his, the sale willaffect only his own share but not those of the other co-owners who did not consent to thesale.This is because under the aforementioned codal provision, the sale or other disposition affects only his undivided share and the transferee gets only what would correspond to his grantor in the partition of the thing owned in common. From the foregoing, it may be deduced that since a co-owner is entitled to sell his undivided share, a sale of the entire property by one co-owner without the consent of the other co-owners is NOT null and void. However, only the rights of the co-owner-seller are transferred, thereby making the buyer a co-owner of the property. Petitioners predecessor-in-interest CAA thus acquired only the rights pertaining to the sellers-heirs of Tito Dignos, which is only undivided share of the two lots. 2. NO. Registered lands cannot be the subject of acquisitive prescription. Petitioners insistence that it acquired the property through acquisitive prescription, if not ordinary, then extraordinary, does not lie. It bears emphasis at this juncture that in the Extrajudicial Settlement and Sale forged by CAA and Tito Dignos heirs the following material portions thereof validate the claim of respondents that the two lots were registered: x x x x That since the OCT of Title of the above-mentioned property/ies has/have been lost and/or destroyed and the VENDEE hereby binds itself to reconstitute said title/s at its own expense and that the HEIRS-VENDORS, their heirs, successors and assigns bind themselves to help in the reconstitution of title so that the said lot/s may be registered in the name of the VENDEE in accordance with law x x x x NOTES: As for petitioners argument that the redemption price should be of the prevailing market value, not of the actual purchase price, since, so it claims, (1) the respondents received just compensation for the property at the time it was purchased by the Government; and, (2) the property, due to improvements introduced by petitioner in its vicinity, is now worth several hundreds of millions of pesos, the law is not on its side. Thus, Article 1088 of the Civil Code provides:

It appears that the two lots were not partitioned by the adjudicatees. It appears further that the heirs of Tito Dignos, who was awarded share in the two lots, sold the entire two lots to the then Civil Aeronautics Administration (CAA) via a public instrument entitled Extrajudicial Settlement and Sale without the knowledge of res pondents whose predecessors-in-interest were the adjudicatees of the rest of the portion of the two lots. In 1996, CAAs successor-in-interest, the Mactan Cebu International Airport Authority (MCIAA), erected a security fence one of the lot and relocated a number of families, who had built their dwellings within the airport perimeter, to a portion of said lot to enhance airport security. MCIAA later caused the issuance in its name of a Tax Declarations of the 2 lots. Respondents soon asked the agents of MCIAA to cease giving third persons permission to occupy the lots but the same was ignored. Respondents thereupon filed a Complaint for Quieting of Title, Legal Redemption with Prayer for a Writ of Preliminary Injunction against MCIAA before the RTC of Lapu-lapu City. Respondents further alleged that neither they nor their predecessors-in-interests sold, alienated or disposed of

Should any of the heirs sell his hereditary rights to a stranger before the partition, any or all of the co-heirs may be subrogated to the rights of the purchaser by reimbursing him for the price of the sale, provided they do so within the period of one month from the time they were notified in writing of the sale by the vendor. The Court may take judicial notice of the increase in value of the lots. As mentioned earlier, however, the heirs of Tito Dignos did not notify respondents about the sale. At any rate, since the Extrajudicial Settlement and Sale stipulates, thus: That the HEIRS-VENDORS, their heirs, assigns and successors, undertake and agree to warrant and defend the possession and ownership of the property/ies herein sold against any and all just claims of all persons whomsoever and should the VENDEE be disturbed in its possession, to prosecute and defend the same in the Courts of Justice. Petitioner is not without any remedy. This decision is, therefore, without prejudice to petitioners right to seek redress against the vendors-heirs of Tito Dignos and their successors-in-interest. Monteroso vs CA Facts: In 1906, Don Fabian married Soledad Doldol. Out of this marriage were bornSoledad, Reygula, Benjamin, and Tirso. On April 8, 1927, Soledad Doldol Monterosopassed away. A little over a year later, Don Fabian contracted a second marriage with SofiaPendejito. From this union were born Florenda, Reynato, Alberto, and Fabian, Jr. Don Fabian filed an intestate proceeding for the estate of his deceased first wife toavoid disputes over the inheritance of his children from his first marriage Land: Parcels F-1 to F-8 (First marriage) and Parcels S-1 to S-4 (Second marriage) The partition in SP No. 309 covered Parcels F-1 to F-5, and adjudicated to DonFabian the whole of Parcels F-1, F-2, and F-3, and one-half of Parcel F-5, while theintestate estate of Soledad D. Monteroso comprised the whole of Parcel F-4 and one-half of Parcel F-5. The intestate estate of Soledad D. Monteroso was partitioned anddistributed to her four children in equal shares On October 26, 1948, Don Fabian also passed away. On July 28, 1969, the children of the late Benjamin D. Monteroso, filed with the RTCa Complaint for Recovery of Property with Damages against their uncle, Tirso D.Monteroso As the heirs of Benjamin alleged in their complaint, their uncle, Tirso, was entrustedwith onefourth portion of Parcel F-4 as part of the share from the estate of SoledadD. Monteroso allotted to their father. However, their uncle refused to surrender anddeliver the same when they demanded such delivery upon their reaching the majorityage. Tirso countered that the portion pertaining to Benjamin was never entrusted to him; itwas in the possession of their sister, Soledad Monteroso-Cagampang, who was notentitled to any share in Parcel F-4, having previously opted to exchange her share insaid property for another parcel of land, i.e., Parcel F-7, then being occupied by her Tirso, in turn, filed a Complaint for Partition and Damages with Receivership ,involving 12 parcels of land against his stepmother, Pendejito, and all his full andhalf-siblings and/or their representatives:(1) the aforementioned 12 parcels of land belong to the conjugalpartnerships of the first and second marriages contracted by Don Fabian;(2) SP No. 309, which purportedly judicially settled the intestateestate of his mother, is null and void for the reason that the project of partitionfailed to comprehend the entire estate of the decedent as Parcels F-6, F-7, and F-8 were excluded, thereby depriving Tirso of his one-fourth share or legitimeover the said three parcels of land; and(3) Parcels S-1 to S-4, having been acquired during the secondmarriage of Don Fabian, are not paraphernal properties of Sofia Pendejito Vda de Monteroso

ISSUE 1.Whether the CA committed reversible error in concluding that, By invoking the benefits of prescription in their favor, the Cagampang spouses are deemed to have admitted the existence of a co-ownership HELD No. What the appellate court tried to convey is clear and simple: partition is the properremedy available to Tirso who is a co-owner of the subject properties by virtue of his being acompulsory heir, like siblings Soledad, Reygula, and Benjamin, of Don Fabian. The right toseek partition is imprescriptible and cannot be barred by laches. Consequently, acquisitiveprescription or laches does not lie in favor of the Cagampang spouses and against Tirso, thegeneral rule being that prescription does not run against a co-owner or co-heir. The onlyexception to the imprescriptibility of an action for partition against a co-owner is when a co-owner repudiates the co-ownership. Thus, the appellate court ruled that by invokingextinctive prescription as a defense, the lone exception against imprescriptibility of action bya co-owner, the Cagampang spouses are deemed to have contextually recognized the co-ownership of Tirso and must have repudiated such coownership in order for acquisitiveprescription to set in. The fact that Tirso and the other compulsory heirs of Don Fabian were excluded from thepossession of their legitime and the enjoyment of the fruits thereof does not per se argue against the existence of a co-ownership By asserting his right as a compulsory heir, Tirso has effectively brought into the open thereality that the Cagampang spouses were holding some of the subject properties in trust andthat he is a co-owner of all of them to the extent of his legal share or legitime thereon Before partition and eventual distribution of Don Fabians intestate estate, a regime of coownership among the compulsory heirs existed over the undivided estate of Don Fabian.Being a co-owner of that intestate estate, Tirsos right over a share thereof is imprescriptible Tirso has at the very least 10 years and at the most 30 years to file the appropriate actionin court. The records show that Tirsos cause of action has not prescribed as he i nstituted anaction for partition in 1970 or only nine years after the considered express repudiation Regarding Parcel S-1, the court ruled that it belongs to all the heirs of Don Fabian and nota paraphernal property of Pendejito as it was under a homestead patent application. Sec 105of CA 141 which governs such provides that the applicant shall be succeeded in his rightsand obligations by his heirs in law after the latter performs all the requirements therefor.Pendejito shall only be entitled to a usufructuary right over the property equal to thecorresponding share of each of the heirs QUIMPO vs. CONSUELO ABAD VDA. DE BELTRAN Eustaquia Perfecto-Abad (Eustaquia) was the owner of several parcels of land Thereafter, Eustaquia died intestate in 1948 leaving these parcels of land to her grandchild and great grandchildren, namely, Joaquin Quimpo and respondents Consuelo, Ireneo, Danilo, Marites, Anita and Helen, all surnamed Abad. Joaquin and respondents undertook an oral partition of parcel III (San Jose property) and parcel IV. Half of the properties was given to Joaquin and the other half to the respondents. However, no document of partition was executed, because Joaquin refused to execute a deed. Consuelo and Ireneo occupied their respective shares in the San Jose property, and installed several tenants over their share in parcel IV. Joaquin, on the other hand, became the administrator of the remaining undivided properties and of the shares of respondents Danilo, Marites, Anita and Helen, who were still minors at that time. Danilo, Marites, Anita and Helen wanted to take possession of the portions allotted to them, but Joaquin prevented them from occupying the same. Joaquin also refused to heed respondents demand for partition of parcels I and II, prompting respondents to file a complaint for judicial partition and/or recovery of possession with accounting and damages with the Regional Trial Court (RTC) of Camarines Sur.3 During the pendency of the case, Joaquin died. Accordingly, he was substituted by his wife, the RTC rendered a Decision5 in favor of respondents, declaring them as co-owners of all the properties left by Eustaquia. It rejected Joaquins claim of absolute ownership over parcels III and IV, and declared void the purported deeds of sale executed by Eustaquia for lack of consideration and consent. On appeal, the CA affirmed the RTC ruling The Quimpos, thus, filed the instant petition for review on certiorari

ISSUE: 2) THE HONORABLE COURT OF APPEALS ERRED IN RULING THAT CO-OWNERSHIP EXISTS AMONG PETITIONERS AND RESPONDENTS OVER THE SUBJECT PARCELS OF LAND; RULING: Likewise, both the trial court and the CA found that Eustaquia was 91 years old, weak and senile, at the time the deeds of sale were executed. In other words, she was already mentally incapacitated by then, and could no longer be expected to give her consent to the sale. The RTC and CA cannot, therefore, be faulted for not giving credence to the deeds of sale in favor of Joaquin. The CA, therefore, committed no reversible error in sustaining the oral partition over parcels III and IV and in invalidating the deeds of sale between Eustaquia and Joaquin. Similarly, we affirm the CA ruling that respondents are co-owners of the subject four (4) parcels of land, having inherited the same from a common ancestor Eustaquia Perfecto-Abad. Petitioners assertion that respondents failed to prove their relationship to the late Eustaquia deserves scant consideration. Consuelo was the grandchild of Eustaquia, while respondents Danilo, Helen, Marites, Anita and also Joaquin Quimpo were Eustaquias great grandchildren. As such, respondents can rightfully ask for the confirmation of the oral partition over parcels III and IV, and the partition of parcels I and II. Jurisprudence is replete with rulings that any co-owner may demand at any time the partition of the common property unless a co-owner has repudiated the co-ownership. This action for partition does not prescribe and is not subject to laches.19 WHEREFORE, the petition is DENIED BETTY LACBAYAN vs. BAYANI S. SAMOY FACTS: Bayani S. Samoy, Jr., despite being married, managed to cohabit and have a child with Betty Lacbayan. During theircohabitation, Samoy and Lacbayan established a man power services and acquired five parcels of land. When theyseparated, they decided to terminate the business and execute a Partition Agreement. However, Lacbayan wantedadditional demands and filed for judicial partition in the Regional Trial Court (RTC) in Quezon City. Samoy, however denied Lacbayans claim of cohabitation and said that the properties were acquired out of his own personal funds without any contribution from Lacbayan. To add, it was alleged that the said properties were registered in Samoy and Lacbayansname to exclude and to prevent the possible dissipation of the said properties from Lacbayans property regime with his wife. ISSUES: I. Whether an action for partition precludes a settlement on the issue of ownership II. Whether the Torrens title over the disputed properties can be collaterally attacked. HELD: I.An action for partition does not preclude a settlement on the issue of ownershipAs held in Municipality of Bian v. Garcia, the determination as to the existence of co ownership is necessary in theresolution of an action for partition. Thus, the first phase of a partition and/or accounting suit is taken up with thedetermination of whether or not a co-ownership in fact exists, and a partition is proper.While it is true that the complaint involved here is one for partition, the same is premised on the existence or non-existence of co-ownership between the parties. Lacbayan insists she is a co-owner pro indiviso of the five real estateproperties based on the transfer certificates of title (TCTs) covering the subject properties. Samoy maintains otherwise.Indubitably, therefore, until and unless this issue of co-ownership is definitely and finally resolved, it would be prematureto effect a partition of the disputed properties. More importantly, the complaint will not even lie if Lacbayan, does noteven have any rightful interest over the subject properties. II.A Torrens certificate of title cannot be collaterally attackedA Torrens certificate of title cannot be collaterally attacked, but that rule is not material to the case at bar. What cannotbe collaterally attacked is the certificate of title and not the title itself. The certificate referred to is that document issuedby the Register of Deeds known as the TCT. In contrast, the title referred to by law means ownership which is, more oftenthan not, represented by that document.Lacbayan confuses title with the certificate of title. Title as a concept of ownership should not be confused with thecertificate of title as evidence of such ownership although both are interchangeably used. Ownership is different from acertificate of title, the latter only serving as the best proof of ownership over a piece of land. The certificate cannot alwaysbe considered as conclusive evidence

of ownership. Needless to say, registration does not vest ownership over aproperty, but may be the best evidence thereof. Tecson vs. Francisco Not innocent buyer for value Indeed, the lack of a plausible explanation why a co-owner would gratuitously cede a very substantial portion of his rightful share to another co-owner in partition renders the foregoing testimonies more credible as against the plain general denial of Atty. Tecson. On this point, We find no reversible error on the part of the Court of Appeals. The established facts have several legal consequences: First. The Second Plan, having been prepared without the knowledge and consent of any of the co-owners of Lot 2189, have no binding effect on them. Second. The Second Partition Agreement is null and void as an absolute simulation,[80] albeit induced by a third party. The fraud perpetrated by Atty. Tecson did more than to vitiate the consent of Waldetrudes and the respondents. It must be emphasized that Waldetrudes and the respondents never had any intention of entering into a new partition distinct from the First Partition Agreement. The established facts reveal that Waldetrudes and the respondents assented to the Second Partition Agreement because Atty. Tecson told them that the instrument was merely required to expedite the sale of Waldetrudes share.[81] In other words, the deceit employed by Atty. Tecson goes into the very nature of the Second Partition Agreement and not merely to its object or principal condition. Evidently, there is an absence of a genuine intent on the part of the co-owners to be bound under a new partition proposing a new division of Lot 2189. The apparent consent of Waldetrudes and the respondents to the Second Partition Agreement is, in reality, totally wanting. For that reason, the Second Partition Agreement is null and void. Third. The Second Partition Agreement being a complete nullity, it cannot be ratified either by the lapse of time or by its approval by the guardianship court.[82] Fourth. The First Plan and the First Partition Agreement remain as the valid and binding division of Lot 2189.Hence, pursuant to the First Partition Agreement, Waldetrudes is the absolute owner of Lot 2189-A with an area of only five hundred seven (507) square meters. Atty. Fausto, on the other hand, has dominion over Lot 2189-B with an area of five hundred eight (508) square meters. Fifth. Inevitably, Waldetrudes can only sell her lawful share of five hundred seven (507) square meters. The sales in favor of Aurora and, subsequently, Atty. Tecson, are thereby null and void insofar as it exceeded the 507 square meter share of Waldetrudes in Lot 2189. Nemo dat quod non habet.[83] Atty. Tecson is not an innocent purchaser for value The remaining bar to the recovery by the respondents of the excess area held by Atty. Tecson is the principle of an innocent purchaser for value of land under the Torrens System of Registration. The petitioners claim that they are bona fidepurchasers of the entire nine hundred sixty-four (964) square meters of land covered by Lot 2189-Bwith Aurora merely relying on the strength of TCT No. T-4,336 in the name of Waldetrudes, while Atty. Tecson placing confidence in TCT No. T4,338 in the name of Aurora.Both TCT Nos. T-4,336 and T-4,338 define the area of Lot 2189-B as nine hundred sixty-four (964) square meters.[84] The petitioners allege that at the time they made their respective purchase, they did not know of the existing partition of Lot 2189 per the First Plan and the First Partition Agreement.[85] We disagree. The proven facts indicate that Atty. Tecson knew or, at the very least, should have known that Atty. Fausto and Waldetrudes were co-owners in equal share of Lot 2189. We must be reminded of the following circumstances: 1. Atty. Tecson was a long-time friend and neighbor of the Faustos.[86] Atty. Tecson himself testified that he considered Atty. Fausto as a good friend and even admitted that he would sometimes visit the latter in his house to play mahjong.[87] By this, Atty. Tecson knew that Atty. Fausto has an actual interest in Lot 2189. 2. Atty. Tecson was the one who presented the Second Partition Agreement to Waldetrudes and the respondents;[88] 3. Waldetrudes and the respondents were not involved in the preparation of the Second Partition Agreement and, at the time they signed the said agreement, had no knowledge of the existence of the Second Plan;[89] and 4. The Second Partition Agreement failed to state the specific areas allotted for each component of Lot 2189 and made no mention of the division proposed by the Second Plan.[90] Being the one behind the execution of the Second Partition Agreement, there is no doubt that Atty. Tecson knew that Lot 2189 was owned in common by Waldetrudes and Atty. Fausto. This, taken together with the instruments unusual silence as to the definite area allotted for each

component lot and the Second Plan, reveals a deliberate attempt on the part of Atty. Tecson to conceal from Waldetrudes and the respondents the unequal division of Lot 2189. The necessity to conceal the disproportionate division of Lot 2189 can only be explained by Atty. Tecsons prior knowledge that such a partition is inherently defective for being contrary to the actual sharing between Waldetrudes and Atty. Fausto. Atty. Tecson is clearly in bad faith. Verily, Atty. Tecson cannot be considered as an innocent purchaser of the excess area of Lot 2189-B.Based on the facts and circumstances prevailing in this case, Atty. Tecson may be charged with actual notice of the defect plaguing the Second Partition Agreement. The respondents may, therefore, recover. Heirs of Albina Ampil v. Sps.Manahan SUMMARY: Exequiel, as representative of the heirs of Albina Ampil filed acomplaint for ejectment against the Manahans. In the complaint, it wasalleged that Albina was the owner of two lots, as evidenced by taxdeclarations, and that Albina had allowed Perfecto and his family to occupya portion of the properties, on the condition that they would vacate thesame, should the need arise. The Manahans filed their answer, averringthat the lots belonged to them, their predecessor-in-interest having been inpeaceful and continuous possession thereof since time immemorial. TheMTC ruled in favor of the Heirs. The RTC affirmed. The CA reversed. The SCupheld the MTC/ RTC. DOCTRINE: In an action for forcible entry and unlawful detainer, any oneof the Art. 487, CC provides that any one of the co-owners may bring anaction for ejectment, without joining the others. The action is not limited toejectment cases but includes all kinds of suits for recovery of possession,because the suit is presumed to have been instituted for the benefit of all. FACTS: Exequiel Ampil, as representative of the heirs of Albina Ampil filed acomplaint for ejectment against the Manahans (Perfecto, Virginia,Teresita, Almario, and Irene). In the complaint, it was alleged that Albina was the owner of two adjoining residential lots in Bulacan, asevidenced by tax declarations, and that Albina had allowed Perfectoand his family to occupy a portion of the properties, on thecondition that they would vacate the same, should the need arise. After the death of Albina, Exequiel and the rest of the heirsrequested Perfecto and family to vacate the property, but the latterrefused. The matter was brought to the Lupong Tagapamayapa, who issued a Certification to File an Action for failure of the parties to amicablysettle their dispute. The Heirs sent a demand letter to the Manahans to surrenderpossession of the lands, but to no avail. So, the Heirs filed acomplaint for ejectment before the MTC. The Manahans filed their answer, averring that the lots belonged tothem, their predecessor-ininterest having been in peaceful andcontinuous possession thereof since time immemorial, and that Albina was never the owner of the property. The MTC rendered judgment in favor of the Heirs, on the basis of thetax declarations. The RTC affirmed. The CA reversed, ruling that tax declarations and receipts are not conclusive proof of ownership or right of possession, and onlybecomes strong evidence of ownership when accompanied by proof of actual possession. The CA denied the Heirs motion for reconsideration. SUBSTANTIVE ISSUE + RATIO:

Who has the better right to the physicalpossession of the disputed property? THE HEIRS.As a rule, petitions for review on certiorari under Rule 45 of the Rules Court are limited only to questions of law and not of fact. The rule, however, admitsof several exceptions. Here, the factual findings of the CA are contrary tothose of the MTC and the RTC. Hence, a review of the case is imperative.In an unlawful detainer case, the physical or material possession of theproperty involved, independent of any claim of ownership by any of theparties, is the sole issue for resolution. But where the issue of ownership israised, the courts may pass upon said issue in order to determine who hasthe right to possess the property. This adjudication, however, is only aninitial determination of ownership for the purpose of settling the issue of possession, the issue of ownership being inseparably linked thereto. As such, the lower courts adjudication of ownership in the ejectment case is merely provisional and would not bar or prejudice an action between the sameparties involving title to the property.In the case at bar, the Court sustains the findings of both the MTC and theRTC. The bare allegation of the Manahans, that they had been in peaceful andcontinuous possession of the lot in question because their predecessor-in-interest had been in possession thereof in the concept of an owner from timeimmemorial, cannot prevail over the tax declarations and otherdocumentary evidence presented by petitioners. In the absence of anysupporting evidence, that of the Heirs deserves more probative value.A perusal of the records shows that Manahans occupation of the lot in question was by mere tolerance. To prove ownership over the property, the Heirs presented the tax declarations covering the properties and acertification issued by the Municipality, showing that their mother, Albina,had been paying the corresponding real property taxes thereon. The Heirs also submitted a s urvey plan, in support of Albinas application for la ndregistration over the disputed lots. In fact, the Registry of Deeds of Bulacanissued Katibayan ng Orihinal na Titulo conferring title over the lot in thenames of the heirs of Albina.Also, one of the Heirs verbally demanded that the Manahans vacate theproperty and when the latter refused, they filed a complaint before theBarangay Lupon. From the minutes of the meeting in the Barangay Lupon, Perfecto admitted Albina allowed them temporary use of the lots andthat they could not leave the premises because they had nowhere else to go.When the parties failed to reach a settlement, the Heirs, in order to protect their rights to the lot in question, filed a case for violation of P.D. No. 772, anAct Penalizing Squatting and other Similar Acts against Perfecto, before theRegional Trial Court. In the said case, Perfecto executed a SinumpaangSalaysay, wherein he admitted that Albina was the owner of the lots inquestion and that he was merely allowed by her to use the property oncondition that they would vacate it on demand. As a result, the court dismissed the complaint because it found out that Perfecto and his familys stay in the questioned lots was lawful because Albina permitted them to usethe lots on the condition that they would vacate the same should Albina needit.On the other hand, the Manahans could not present proof that they and theirpredecessors-in-interest had openly and continuously possessed the subject land since time immemorial. Granting that the Manahans or theirpredecessors-in-interests had been in possession in the concept of an ownersince time immemorial, none of them declared the disputed lots for taxationpurposes and, thus, never paid taxes thereon. The Manahans' allegation that they were in peaceful, continuous and adverse possession of the lots inquestion, unsupported by any evidence, is not substantial to establish theirinterest over the property.Well established is the rule that ownership over the land cannot be acquiredby mere occupation. While it is true that tax declarations are not conclusiveevidence of ownership, they, nevertheless, constitute at least proof that theholder has a claim of title over the property. It strengthens one's bona fideclaim of acquisition of ownership. PROCEDURAL ISSUE + RATIO: Does Exequiel have authority to file thecomplaint on behalf of his co-heirs? YES.Art. 487, CC provides that any one of the co-owners may bring an actionfor ejectment, without joining the others. The action is not limited toejectment cases but includes all kinds of suits for recovery of possession,because the suit is presumed to have been instituted for the benefit of all. Figuracion v Gerilla The Facts The parties are the heirs of Leandro Figuracion (Leandro) who died intestate in May 1958. Petitioner Carolina is the surviving spouse. The other petitioners Elena Figuracion-Ancheta, Hilaria A. Figuracion (Hilaria), Felipa Figuracion-Manuel (Felipa), Quintin Figuracion, and Mary Figuracion-Ginez and respondent Emilia were Carolina and Leandros children.4 Subject of the dispute are two parcels of land both situated in Urdaneta, Pangasinan, which were acquired by Leandro during his lifetime. Both lands were registered in the name of "Leandro Figuracion married to Carolina Adviento". Leandro executed a Deed of Quitclaim over the above real properties in favor of his six (6) children on August 23, 1955. Their shares, however, were not

delineated with particularity because spouses Leandro and Carolina reserved the lots and its fruits for their expenses. Also involved in the controversy is Lot No. 707 of the Cadastral Survey of Urdaneta, Pangasinan, with an area of 3,164 square meters originally owned by Eulalio Adviento (Eulalio), covered by Original Certificate of Title (OCT) No. 15867 issued in his name on August 21, 1917. Eulalio begot Agripina Adviento (Agripina) with his first wife Marcela Estioko (Marcela), whom Eulalio survived. When he remarried, Eulalio had another daughter, herein petitioner Carolina, with his second wife, Faustina Escabesa (Faustina).6 On November 28, 1961, Agripina7 executed a Deed of Quitclaim8 over the eastern half of Lot No. 707 in favor of her niece, herein respondent Emilia. petitioner Carolina executed an Affidavit of Self-Adjudication9adjudicating unto herself the entire Lot No. 707 as the sole and exclusive heir of her deceased parents, Eulalio and Faustina.10 On the same date, Carolina also executed a Deed of Absolute Sale11 over Lot No. 707 in favor of petitioners Hilaria and Felipa, who in turn immediately caused the cancellation of OCT No. 15867 and the issuance of TCT No. 42244 in their names.12 In 1971, Emilia and her family went to the United States and returned to the Philippines only in 1981. Upon her return and relying on the Deed of Quitclaim, she built a house on the eastern half of Lot No. 707.13 The legal debacle of the Figuracions started in 1994 when Hilaria and her agents threatened to demolish the house of Emilia who, in retaliation, was prompted to seek the partition of Lot No. 707 as well as Lot Nos. 2299 and 705. On May 23, 1994, respondent Emilia instituted the herein Complaint15 for the partition of Lot Nos. 2299, 705 and 707, annulment of the Affidavit of SelfAdjudication, Deed of Absolute Sale and TCT No. 42244, reconveyance of eastern half portion of Lot No. 707, quieting of title and damages. ISSUE: whether or not respondent Emilia is the owner of the eastern half portion of Lot No. 707 RULING: Mere issuance of a certificate of title in the name of any person does not foreclose the possibility that the real property may be under coownership with persons not named in the certificate, or that the registrant may only be a trustee, or that other parties may have acquired interest over the property subsequent to the issuance of the certificate of title.36 Stated differently, placing a parcel of land under the mantle of the Torrens system does not mean that ownership thereof can no longer be disputed. The certificate cannot always be considered as conclusive evidence of ownership.37 In this case, co-ownership of Lot No. 707 was precisely what respondent Emilia was able to successfully establish, as correctly found by the RTC and affirmed by the CA. The status of Agripina and Carolina as the legitimate heirs of Eulalio is an undisputed fact. As such heirs, they became co-owners of Lot No. 707 upon the death of Eulalio on July 20, 1930. Since Faustina was predeceased by Eulalio, she likewise became a co-owner of the lot upon Eulalios death. Faustinas share, however, passed on to her daughter Carolina when the former died on October 18, 1949. The Affidavit of Self-Adjudication executed by Carolina did not prejudice the share of Agripina because it is not legally possible for one to adjudicate unto himself an entire property he was not the sole owner of. A co-owner cannot alienate the shares of her other coowners nemo dat qui non habet.38 Hence, Lot No. 707 was a co-owned property of Agripina and Carolina. As co-owners, each of them had full ownership of her part and of the fruits and benefits pertaining thereto. Each of them also had the right to alienate the lot but only in so far as the extent of her portion was affected. Thus, when Carolina sold the entire Lot No. 707 on December 11, 1962 to Hilaria and Felipa without the consent of her co-owner Agripina, the disposition affected only Carolinas pro indiviso share, and the vendees, Hilaria and Felipa, acquired only what corresponds to Carolinas share. A co-owner is entitled to sell his undivided share; hence, a sale of the entire property by one co-owner without the consent of the other co-owners is not null and void and only the rights of the co-owner/seller are transferred, thereby making the buyer a co-owner of the property. Accordingly, the deed of sale executed by Carolina in favor of Hilaria and Felipa was a valid conveyance but only insofar as the share of Carolina in the co-ownership is concerned. As Carolinas successors-in-interest to the property, Hilaria and Felipa could not acquire any superior right in the property than what Carolina is entitled to or could transfer or alienate after partition. In a contract of sale of co-owned property, what the vendee obtains by virtue of such a sale are the same rights as the vendor had as co-owner, and the vendee merely steps into the shoes of the vendor as co-owner. Hilaria and Felipa did not acquire the undivided portion pertaining to Agripina, which has already been effectively bequeathed to respondent Emilia as early as November 28, 1961 thru the Deed of Quitclaim. In turn, being the successor-in-interest of

Agripinas share in Lot No. 707, respondent Emilia took the formers place in the co -ownership and as such co-owner, has the right to compel partition at any time Possession in the concept of a holder Aranda v. Republic Under the Regalian doctrine which is embodied in Section 2, Article XII of the 1987 Constitution, all lands of the public domain belong to the State, which is the source of any asserted right to ownership of land. All lands not appearing to be clearly within private ownership are presumed to belong to the State. Unless public land is shown to have been reclassified or alienated to a private person by the State, it remains part of the inalienable public domain. To overcome this presumption, incontrovertible evidence must be established that the land subject of the application is alienable or disposable. To prove that the land subject of an application for registration is alienable, an applicant must establish the existence of a positive act of the government such as a presidential proclamation or an executive order; an administrative action; investigation reports of Bureau of Lands investigators; and a legislative act or a statute. The applicant may also secure a certification from the Government that the lands applied for are alienable and disposable. ARANDA vs. REPUBLIC FACTS: Subject of a petition for original registration before the RTC is a parcel of land situated in San Andres, Malvar, Batangas with an area of 9,103 square meters and designated as Lot 3730, Psc 47, Malvar Cadastre. The petition4 was originally filed by ICTSI Warehousing, Inc. (ICTSI-WI) represented by its Chairman, Enrique K. Razon, Jr. The Republic through the Office of the Solicitor General (OSG) filed its opposition5 on grounds that the land applied for is part of the public domain and the applicant has not acquired a registrable title thereto under the provisions of Commonwealth Petitioner argues that the deeds of confirmation of the 1946 sale in favor of Anatalio Aranda and the 1965 donation to petitioner are competent proof of transfer of ownership notwithstanding that these were executed only in the year 2000. He asserts that the testimonies of witnesses Merlita Aranda-Enriquez and Luis Olan on the fact of loss and destruction of copies of the aforesaid deeds constitute secondary evidence of the contents thereof based on recollection of persons who are adversely affected. Such testimonial evidence coupled with the deeds of confirmation warrants the application of the exception from the best evidence rule. Petitioner thus contends that the CA had no legal basis to doubt the veracity of the donation and sale of the subject property, and to conclude that the confirmation deeds can be treated as compromise agreement considering that the transactions had been previously completed and perfected by the parties. ISSUE: this appeal by way of a petition for review on certiorari under Rule 45 alleging that the decision of the CA is based on a misapprehension of facts with regard to compliance with the required 30 years of open, exclusive, public and adverse possession in the concept of owner. RULING: Petitioner has not explained the discrepancies in the dates of classification20 mentioned in the foregoing government certifications. Consequently, the status of the land applied for as alienable and disposable was not clearly established. We also agree with the CA that petitioners evidence failed to show that he possessed the property in the manner and for the duration required by law. Petitioner presented tax declarations and the deeds of confirmation of the 1946 sale from the original owner (Lucio Olan) to Anatalio Aranda and the 1965 donation made by the latter in favor of petitioner. But as found by the CA, the history of the land shows that it was declared for taxation purposes for the first time only in 1981. On the other hand, the Certification issued by the Municipal Treasurer of Malvar stated that petitioner, who supposedly received the property from his father in 1965, had been paying the corresponding taxes for said land "for more than five consecutive years including the current year [1999]," or beginning 1994 only or just three years before the filing of the application for original registration. While, as a rule, tax declarations or realty tax payments of property are not conclusive evidence of ownership, nevertheless they are good indicia of possession in the concept of owner, for no one in his right mind would be paying taxes for a property that is not in his actual or constructive possession they constitute at least proof that the holder has a claim of title over the property. Petitioner likewise failed to prove the alleged possession of his predecessors-in-interest. His witness Luis Olan testified that he had been visiting the land along with his father Lucio since he was 6 years old (he was 70 years old at the time he testified), or as early as 1936. Yet, there was no evidence that Lucio Olan declared the property for tax purposes at anytime before he sold it to Anatalio Aranda. There is also no

showing that Anatalio Aranda declared the property in his name from the time he bought it from Lucio Olan. And even assuming that Lucio actually planted rice and corn on the land, such statement is not sufficient to establish possession in the concept of owner as contemplated by law. Mere casual cultivation of the land does not amount to exclusive and notorious possession that would give rise to ownership. Specific acts of dominion must be clearly shown by the applicant. Concept of Owner DECALENG vs.BISHOP FACTS: the Philippine Episcopal Church (PEC), is a religious corporation duly organized and registered under the laws of the Republic of the Philippines, performing mission work in over 500 communities throughout the country PEC-EDNP filed before the Regional Trial Court (RTC) of Bontoc, Mountain Province, Branch 36, a Complaint for Accion Reinvindicatoria and Accion Publiciana against Ambrosio Decaleng and Fabian Lopez PEC-EDNP alleged that it is the owner of two parcels of land in the Municipality of Sagada, located in areas commonly known as Ken-geka and Ken-gedeng. ISSUE: THE HONORABLE COURT OF APPEALS SERIOUSLY ERRED IN HOLDING THAT RESPONDENT HAS ESTABLISHED ITS OWNERSHIP AND POSSESSION OVER THE LOTS IN DISPUTE, NOTWITHSTANDING THE FINDING OF THE TRIAL COURT THAT SAID LOTS WERE POSSESSED AND OCCUPIED BY THE PETITIONERS AND THEIR PREDECESSORS IN INTEREST RULING: An accion reinvindicatoria is an action to recover ownership over real property.42 Article 434 of the New Civil Code provides that to successfully maintain an action to recover the ownership of a real property, the person who claims a better right to it must prove two things: first, the identity of the land claimed by describing the location, area, and boundaries thereof; and second, his title thereto.43 The Court finds that PEC-EDNP was able to successfully prove both requisites by preponderance of evidence, both documentary and testimonial. It is apt to observe that actual possession of an owner did not need to be the actual and physical possession and occupation of every inch or portion of the property. That is an impossibility. Constructive possession is sufficient, for, according to Ramos v. Director of Lands: "The claimant has color of title; he acted in good faith; and he has had open, peaceable, and notorious possession of a portion of the property, sufficient to apprise the community and the world that the land was for his enjoyment. (See Arts. 446, 448, Civil Code.) Possession in the eyes of the law does not mean that a man has to have his feet on every square meter of ground before it can be said that he is in possession. x x x" The spouses Decaleng were similarly vague as to the basis of their title. The evidence for the spouses Decaleng do not establish how their predecessors-in-interest acquired the disputed properties and how long they and their predecessors-in-interest have been in possession of the same. While the spouses Decaleng testified that they inherited the properties in Ken-geka and Kengedeng from their parents who, in turn, inherited the same from their own parents, there still remains the question as to how the spouses Decalengs predecessors -in-interest originally came into possession of the subject properties. Neither can the spouses Decaleng claim imperfect title to the properties in Ken-geka and Kengedeng for such can only be acquired by possession of lands of the public domain for the period required by law.69 Because the spouses Decaleng failed to provide and prove the necessary details on how and when their predecessors-in-interest came to possess the disputed properties, there is no way for the Court to determine whether or not said properties were still part of the public domain when occupied by the spouses Decalengs predecessors -in-interest. As the Court previously found herein, the Ken-geka property was already covered by a Certificate of Title issued in the name of the U.S. Episcopal Church (the predecessor-in-interest of PEC-EDNP) on February 18, 1915 and the Ken-gedeng property had been in the possession under claim of title by the U.S. Episcopal Church ever since its arrival in the Mountain Province in 1901. WHEREFORE, the Petition of the spouses Decaleng in G.R. No. 171209 is hereby DENIED for lack of merit.

Abobon vs. Abobon Property; possession as right of the owner. It is beyond question under the law that the owner has not only the right to enjoy and dispose of a thing without other limitations than those established by law, but also the right of action against the holder and possessor of the thing in order to recover it. He may exclude any person from the enjoyment and disposal of the thing, and, for this purpose, he may use such force as may be reasonably necessary to repel or prevent an actual or threatened unlawful physical invasion or usurpation of his property FACTS: The controversy involves the rightful possession of a parcel of registered land. The respondents, who were the registered owners, sued the petitioner, their first cousin, to recover 1the possession of the land in question, stating that they had only allowed the petitioner to use the land out of pure benevolence, but the petitioner asserted that the land belonged to him as owner by right of succession from his parents ISSUE: WHETHER OR NOT THE COURT OF APPEALS GRAVELY ERRED IN AWARDING POSSESSION OF SUBJECT PREMISES TO RESPONDENTS WITHOUT CITING ANY REASONS THEREFOR AND DESPITE THE FACT THAT EVIDENCE ON HAND SHOWS PETITIONER BECAME THE LAWFUL OWNER THEREOF PRIOR TO TIME RESPONDENTS ACQUIRED THE SAME RULING: First of all, a fundamental principle in land registration under the Torrens system is that a certificate of title serves as evidence of an indefeasible and incontrovertible title to the property in favor of the person whose name appears therein.12 The certificate of title thus becomes the best proof of ownership of a parcel of land;13 hence, anyone who deals with property registered under the Torrens system may rely on the title and need not go beyond the title.14 This reliance on the certificate of title rests on the doctrine of indefeasibility of the land title, which has long been wellsettled in this jurisdiction. It is only when the acquisition of the title is attended with fraud or bad faith that the doctrine of indefeasibility finds no application.15 Accordingly, we rule for the respondents on the issue of the preferential right to the possession of the land in question. Their having preferential right conformed to the age-old rule that whoever held a Torrens title in his name is entitled to the possession of the land covered by the title.16 Indeed, possession, which is the holding of a thing or the enjoyment of a right,17 was but an attribute of their registered ownership. It is beyond question under the law that the owner has not only the right to enjoy and dispose of a thing without other limitations than those established by law, but also the right of action against the holder and possessor of the thing in order to recover it.18 He may exclude any person from the enjoyment and disposal of the thing, and, for this purpose, he may use such force as may be reasonably necessary to repel or prevent an actual or threatened unlawful physical invasion or usurpation of his property. Casimiro Development v Mateo (inulit) Good Faith The Heirs of Nicolas S. Cabigas vs. Melba L. Limbaco FACTS: Petitioners purchased 2 lots from Cobarde in 1980 who in turn had purchased these lots from Ouano in 1948. Nothwithstanding the sale, the 2 lots remained registered in the name of Ouano. Ouano was able to sell these same lots to the National Airports Corporation (NAC) in 1952 for its airport expansion project. NAC promptly had the titles for these properties registered in its name. When the airport expansion project fell through, respondent and the legal heirs of Ouano succeeded in reclaiming title to the 2 lots through an action for reconveyance. The titles over these lots were thereafter registered in their names then they subdivided the lots and sold them to various buyers who registered the titles over their respective lots in their names.

ISSUE: Who has a better right to the subject lots. HELD: Cabigas spouses are not buyers in good faith. A purchaser in good faith is one who buys the property of another without notice that some other person has a right to or interest in such property, and pays a full and fair price for the same at the time of such purchase or before he has notice of the claim of another person. It is a well-settled rule that a purchaser cannot close his eyes to the fact which should put a reasonable man upon his guard, and then claim that he acted in good faith under the belief that there was no defect in the title of the vendor. At the time of the sale to the Cabigas spouses, the land was registered not in Cobardes name but in Ouanos name. They relied completely onCobardes representation that he owned the properties in question and did not even bother to perform the most perfunctory of investigations by checking the properties titles with the Registry of Deeds. Their failure to exercise the plain common sense expected of real estate buyers bound them to the consequences of their own inaction. The overriding consideration to determine ownership of an immovable property is the good or bad faith of the buyer, specifically who first registered the ale with the Registry of Deeds in good faith. Since the Cabigas spouses have no legally recognizable interest in the lots in question, it follows that the petitioners, who are subrogated to the rights of the former by virtue of succession, also have no legally recognizable rights to the properties that could be enforced by law. Applicable presumptions Loreto Luga vs. Sps. Elena and Rogelio Arciaga FACTS: The suit concerns a parcel of land where petitioner became a tenant and occupied the subject parcel since 1957, eventually building a house of light materials thereon. In 1970, a Deed of Transfer of Right was executed in favor of Rogelio who paid the sum of 10k as consideration for the improvements thereon. Elena applied for a patent/title over the subject parcel. Posting of a notice was caused directing persons affected thereby to make known their adverse claim, if any. In 1988, the application was approved, a Deed of Absolute Sale was executed and a certificate of title was issued in favor of Elena, who forthwith, declared the land in her name for taxation purposes and started paying the real estate taxes due thereon. In 1994, Loreto filed a complaint for reconveyance of title and damages against respondents, claiming the he had been in possession of the subject parcel since 1957 and that he discovered the titling of the same in the name of Elena only in 1993. Evidence on record disclosed that petitioner was merely allowed to occupy the land and never possessed the parcel in the concept of an owner. He never declared the land for taxation purposes in his own name and only attempted to do so only in 1993. Since Loretos evidence does not constitute the well-nigh incontrovertible evidence required to acquire title to land through possession and occupation, he is not entitled to thereconveyance. ISSUE: WON petitioner is entitled to reconveyance of the property. HELD: Since possession may be exercised in ones own name or in that of another and it is not necessary for the owner or holder of the thing to personally exercise his possessory rights. Loretos tolerated occupancy of the land cannot be said to have ousted the possession claimed by the Sps. Arciaga. An action for reconveyance of property is, after all, a legal and equitable remedy available to landowner whose property has been wrongfully or erroneously registered in anothers name, after 1 year from the date of the decree of registration so long as the property has not passed to an innocent purchaser for value. The decree of registration is respected as incontrovertible. Where there is a wrongful or erroneous registration in another persons name, the rightful owner or one with a better right can seek reconveyance of the property and cancellation of title. Loreto failed to prove a right than petitioners over the land. Acquisition and loss of possession

Republic of the Philippines vs. Zenaida Guinto-Aldana FACTS: Respondents filed an application for registration of title over 2 pieces of land, professing themselves to be co-owners of these lots having acquired them by succession from their predecessors. That until the time of the application, they and their predecessors-in-interest have been in actual, open, peaceful, adverse, exclusive and continuous possession of these lots in the concept of an owner and that they had consistently declared the property in their name for purposes of real estate taxation. In support of their application, respondents submitted to the court the pertinent tax declarations, together with the receipts of payment thereof. Petitioner opposed the application for the reason that the tax declaration submitted to the court did not constitute competent and sufficient evidence of bona fide acquisition in good faith or of prior possession in the concept of an owner. ISSUE: WON respondents have occupied and possessed the property openly, continuously, exclusively and notoriously under a bona fide claim of ownership. HELD: Respondents possession through their predecessors-in-interest dates back to as early as 1937 when the property had already been declared for taxation by respondents father. Respondents could have produced more proof of this kind had it not been for the fact that, the relevant portions of the tax records on file with the Provincial Assessor had been burned when its office was razed by fire in 1997. With the tax assessments therecame next tax payments. Respondents receipts for tax expenditures were likewise in therecords and in these documents the predecessors of respondents were the named owners of the property. Tax declarations and realty tax payment are not conclusive evidence ofownership, nevertheless, they are a good indication of possession in the concept of an owner. No one in his right mind would be paying taxes for a property that is not in his actual or at least constructive possession. Indeed, respondents herein have been in possession of the land in the concept of an owner, open, continuous, peaceful and without interference and opposition from the government or from any private individual. Itself makes their right thereto unquestionably settled and hence, deserving of protection under the law. Special Cases of Usufruct Fabie v. David FACTS: Josefa Fabie is the usufructuary of the income of certain houses located at 372-376 Santo Cristo, Binondo, and 950-956 Ongpin, Santa Cruz, Manila, under the ninth clause of the will of the deceased Rosario Fabie y Grey. The owner of Santo Cristo property is the respondent Juan Grey. Litigation arose between Josefa Fabie as plaintiff and Juan Grey as defendant and the owner of the Ongpin property as intervenors, involving the administration of the houses mentioned in clause 9 of the will referred to above. In June 1945 Josefa Fabie commenced an action of unlawful detainer against Ngo Boo Soo (who says that his correct name is Ngo Soo), alleging that the defendant is occupying the premises located at 372-376 Santo Cristo on a month-to month rental payable in advance not later than the 5th of each month; that she is the administratrix and usufructuary of said premises; that the defendant offered to pay P300 monthly rent payable in advance not later than the 5th of every month, beginning the month of April 1945, for the said of premises including the one door which said defendant, without plaintiffs consent and contrary to their agreement, had subleased to another Chinese, but plaintiff refused, based on the fact that the plaintiff very badly needs the said house to live in, as her house was burned by the Japanese on the occasion of the entry of the American liberators in the City; that defendant was duly notified to leave the said premises, but he refused; and she prayed for judgment of eviction and for unpaid rentals. The defendant answered alleging that he was and since 1908 had been a tenant of the premises in question, which he was using and had always used principally as a store and secondarily for living quarters; that he was renting it from its owner and administrator Juan Grey; that plaintiff is merely the usufructuary of the income therefrom, and by agreement between her and said owner, her only right as usufructuary of the income is to receive the whole of such income; that she has no right or authority to eject tenants, such right being in the owner and administrator of the house, Juan Grey; that plaintiff has never had possession of said property; that defendants lease

contract with the owner of the house is for 5-year period, with renewal option at the end of each period, and that his present lease due to expire on December 31, 1945; that on June 1, 1945, defendant made a written offer to plaintiff to compromise and settle the question of the amount of rent to be paid by defendant but said plaintiff rejected the same for no valid reason whatever and instituted the present action; that the reason plaintiff desires to eject defendant from the property is that she wishes to lease the same to other persons for a higher rent, ignoring the fact that as usufructuary of the income of the property she has no right to lease the property. ISSUE: Who is entitled to administer the property subject matter of this case and who should be the tenant? HELD: The usufructuary has the right to administer the property in question. All the acts of administration to collect the rents for herself, and to conserve the property by making all necessary repairs and paying all the taxes, special assessments, and insurance premiums thereon were by court judgment vested in the usufructuary. The pretension of the respondent Juan Grey that he is the administrator of the property with the right to choose the tenants and to dictate the conditions of the lease is contrary to both the letter and the spirit of the said clause of the will, the stipulation of the parties, and the judgment of the court. He cannot manage or administer the property after all the acts of management and administration have been vested by the court, with his consent, in the usufructuary. He admitted that before said judgment he had been collecting the rents as agent of the usufructuary under an agreement with the latter. As long as the property is properly conserved and insured he can have no cause for complaint, and his right in that regard is fully protected by the terms of the stipulation and the judgment of the court above mentioned. To permit him to arrogate to himself the privilege to choose the tenant, to dictate the conditions of the lease, and to sue when the lessee fails to comply therewith, would be to place the usufructuary entirely at his mercy. It would place her in the absurd situation of having a certain indisputable right without the power to protect, enforce, and fully enjoy it. BALURAN V. NAVARRO DOCTRINE: The manner of terminating the right of usufruct may be stipulated by the parties such as in this case, the happening of a resolutory condition. FACTS: Baluran and Paraiso (ancestor of Obedencio) entered into a contract which they called barter, but in fact stipulated that they would only transfer the material possession of their respective properties to each other. Thus, Baluran will be allowed to construct a residential house on the land of Paraiso while Paraiso is entitled to reap the fruits of the riceland of Baluran. The contract prohibited them from alienating the properties of the other and contained a stipulation that should the heirs of Paraiso desire to re-possess the residential lot, Baluran is obliged to return the lot. Indeed, years after, Obedencio (grandchild of Paraiso) acquired the ownership of the residential lot from his mother and demanded that Baluran, who was in possession, vacate. Baluran now counters that the barter already transferred ownership. ISSUE: W/N the contract was a barter or something else (hula ko lang usufruct) RULING: IT IS USUFRUCT. First, the contract is what the law defines it to be and not what the parties call it. It is very clear that what the parties exchanged was not ownership, but merely material possession or the right to enjoy the thing. Now, because it is usufruct, the law allows the parties to stipulate the conditions including the manner of its extinguishment. In this case, it was subject to a resolutory condition which is in case the heir of Paraiso (a third party) desires to repossess the property. Upon the

happening of the condition, the contract is extinguished. Therefore, Baluran must return the land to Obedencia. But since Art. 579 allows the usufructuary to remove improvements he made, Baluran may remove the house he constructed. One last point. At the time of this case, the Obedencias were also in possession of the riceland of Baluran. Although it was not proper to decide the issue of possession in this case, the Court nevertheless decided on the matter and order the Obedencias to vacate the property inasmuch as there was an extinguishment of a reciprocal obligations and rights GABOYA V. CUI FACTS: Don Mariano sold his three lots pro diviso to his three children. One of his children, due to lack of funds, was not able to purchase part of the land. This reverted back to the father. As part of the sale, the father reserved for himself the usufruct of the property. He coowned the land with his children then. A building was then constructed in a portion of the land, wherein the rentals was given to the father. Thereafter, he two children who were co-owners obtained a loan, secured by a mortgage, with authority of the father, to construct a commercial building. The father alleges that since he has a usufruct over the land, he has usufruct or share in the rentals earned through the constructed building. ISSUE: W/N the usufruct extended to the rentals of the building subsequently built by the 2 children co-owners HELD: NO. The reserved right of vendor on a parcel of land does NOT include rentals from the building subsequently constructed on the vacant lots, but that it did entitle the usufructuary (Don Mariano) to a reasonable rent for the portion of the land being occupied by the building. Extinguishment of the usufruct CALINGASAN vs.RIVERA The Factual Antecedents During their lifetime, respondent Wilfredo Rivera and his wife, Loreto Inciong, acquired several parcels of land. On July 29, 1982, Loreto died, leaving Wilfredo and their two daughters, Evangeline and Brigida Liza, as her surviving heirs.7 About eleven (11) years later, or on March 29, 1993, Loretos heirs executed an extrajudicial settlement of her one-half share of the conjugal estate, adjudicating all the properties in favor of Evangeline and Brigida Liza; Wilfredo waived his rights to the properties, with a reservation of his usufructuary rights during his lifetime Almost a decade later, or on March 13, 2003,10 Wilfredo filed with the Municipal Trial Court in Cities (MTCC) of Lipa City a complaint for forcible entry against the petitioners and Star Honda, Inc., docketed as Civil Case No. 0019-03. Wilfredo claimed that he lawfully possessed and occupied the two (2) parcels of land located along C.M. Recto Avenue, Lipa City, Batangas, covered by TCT Nos. T-87494 and T-87495, with a building used for his furniture business. Taking advantage of his absence due to his hospital confinement in September 2002, the petitioners and Star Honda, Inc. took possession and caused the renovation of the building on the property. In December 2002, the petitioners and Star Honda, Inc., with the aid of armed men, barred him from entering the property ISSUE: Wilfredos death did not render moot the forcible entry case.

RULING: To go back to basics, the petition before us involves the recovery of possession of real property and is a real action that is not extinguished by the death of a party. The judgment in an ejectment case is conclusive between the parties and their successors-in-interest by title subsequent to the commencement of the action; hence, it is enforceable by or against the heirs of the deceased.1wphi1 This judgment entitles the winning party to: (a) the restitution of the premises, (b) the sum justly due as arrears of rent or as reasonable compensation for the use and occupation of the premises, and (c) attorneys fees and costs. The complicating factor in the case is the nature and basis of Wilfredos possession; he was holding the property as usufructuary, although this right to de jure possession was also disputed before his death, hand in hand with the de facto possession that is subject of the present case. Without need, however, of any further dispute or litigation, the right to the usufruct is now rendered moot by the death of Wilfredo since death extinguishes a usufruct under Article 603(1) of the Civil Code. This development deprives the heirs of the usufructuary the right to retain or to reacquire possession of the property even if the ejectment judgment directs its restitution. WHEREFORE, we hereby DENY the appeal Conditions Not Affecting Usufruct VDA DE ALBAR V. CARANDANG, 106 PHIL 855- USUFRUCT The reparation or indemnity given in exchange for the destruction of the building is the substitute for the building itself. The indemnity is the capital which belongs to the naked owner while the interest on the capital is the fruits which belong to the usufructuary. Notes: The Civil Code contemplates a situation where the owner pays for the construction of a new building. However, the twist in this case was that the naked owner did not have to construct a new one because the Chinaman had one built at his own expense. This is the reason why the court had a difficult time ascertaining who had the right to the indemnity given by the government for the destruction of the building due to the war. Majority Opinion: The reparation should be treated as fruits. Usufructuary should get 6% of the reparation (from the time it was actually received to the tend of the life of the usufruct) because it was not used to construct a new building. Otherwise, the naked owner is enriched twice first from the reparation and second from the fruits if payment of rent stops when the building is constructed. Thus, the new building should be considered as the capital, and the reparation as fruits. The naked owner should share the reparation with the usufructuary to prevent unjust enrichment. Ponentes Opinion: The reparation should be treated as capital (NCC). Only the interest on the reparation up to the date that the new building was constructed time should be given to the usufructuary, not the interest until the end of the usufruct. The reparation itself which is considered as the capital (which rightfully belongs to the naked owner) intended to replace the old building. The intention was fulfilled when the Chinaman constructed a new one. It was not the naked owners fault that he need not use the re paration to construct a new building. The owner is not doubly compensated because it was not his fault. Dissenting opinion: There is double benefit in either case. In the first, the usufructuary will be receiving interest on the reparation and rent from the building. In the second, the naked owner receives value for the building and the construction of a building at no expense to him. Payment of interest should continue during the life of the usufruct (not just 6%) because the war damage is the equivalent to the building. The construction of the new building does not relieve the owners of the land used in the war damage payment from continuing the payment of interest. If they had used it to construct the building, they would have been freed from paying interest but they did not. FACTS: Doa Rosario Fabie y Grey was the owner of the lot in the City of Manila with a building and improvements, and by a will left by her upon her death which was duly probated she devised the naked ownership of the whole property to Rosario Grey Vda. de Albar, et al. but its usufruct to Josefa Fabie for life. During liberation, as a consequence of the fire that gutted the building in many portions of Manila, the building on the Ongpin lot was burned, leaving only the walls and other improvements that were not destroyed by the fire.

One Au Pit, a Chinaman, offered to lease the property for a period of five years, at the same time agreeing to construct on the lot a new building provided the naked owners as well as the usufructuary sign the agreement of the lease. As the usufructuary maintains that she has the exclusive right to cede the property by lease and to receive the full rental value by virtue of her right to usufruct while on the other hand the naked owners maintain that the right of usufruct was extinguished when the building was destroyed, the right of the usufructory being limited to the legal interest on the value of the lot and the materials, in order that the agreement of lease may be affected, the parties agreed on a temporary compromise whereby the naked owners would receive P100.00, or 20% of the monthly rental of P500.00 and the usufructuary the balance of 80% or P400.00 of said monthly rental. It was likewise stipulated in the agreement that the title to the building to be constructed would accrue to the land upon it completion as an integral part of the lot covered by the transfer certificate of title issued in the name of the naked owners but subject to the right of usufruct of Josefa Fabie. The parties expressly reserved the right to litigate their respective claims after the termination of the contract of lease to determine which of said claims was legally correct. By reason of the destruction of the building on the Ongpin property, the United States War Damage Commission approved the claim that was presented for the damage caused to the property, paid to and received by the naked owners. In the meantime, the usufructuary paid the real estate taxes due on the property at Ongpin for the years 1945 to 1952. ISSUE: Whether or not the usufruct included the building and the land? W/N the usufructuary (FABIE) or naked owner (VDA DE ALBAR) should undertake the reconstruction? W/N the usufructuary should pay the real estate taxes? HELD: The usufruct for life extended to the land and the building. From the above, it is clear that when the deceased constituted the life usufruct on the rentals "fincas situadas" in Ongpin and Sto. Cristo streets, she meant to impose the encumbrance both the building and the land on which it is erected for indeed the building cannot exist without the land. And as this Court well said, "The land, being an indispensable part of the rented premises cannot be considered as having no rental value whatsoever." Moreover, in the Spanish language, the term "fincas" has a broad scope; it includes not only building but land as well. (Diccionario Ingles-Espaol, por Martines Amador) Since only the building was destroyed and the usufruct is constituted not only on the building but on the land as well, then the usufruct is not deemed extinguished by the destruction of the building for under the law usufruct is extinguished only by the total loss of the thing subject of the encumbrance (Article 603, old Civil Code). FABIE, the usufructuary has the discretion to reconstruct the building. Of course, this is addressed to the wisdom and discretion of the usufructuary who, to all intents and purposes is deemed as the administrator of the property. This has been clarified in the case of Fabie vs. Gutierrez David, 75 Phil., 536, which was litigated between the same parties and wherein the scope of the same provision of the will has been the subject of interpretation. The usufructuary should pay the taxes. We find, however, merit in the contention that the real estate taxes paid by respondent in her capacity as usufractuary for several years previous to the present litigation should be paid by her, as she did, instead of by petitioners not only because she bound herself to pay such taxes in a formal agreement approved by the court in Civil Case No. 1569 of the Court of First Instance of Manila (Fabie vs. Gutierrez David, supra). In the case, which involved the same parties and the same properties subject to usufruct, the parties submitted an amicable agreement which was approved by the court wherein the usufructuary, herein respondent, bound herself to pay all the real estate taxes, special assessment and insurance premiums, and make all the necessary repairs on each of the properties covered by the usufruct and in accordance with said agreement, respondent paid all the taxes for the years 1945 to 1954.

Vous aimerez peut-être aussi